You are on page 1of 70

Solutions Manual for Joseph Taylors

Foundations of Analysis
Michael Senter

Part I
Single Variable - 3210

Chapter 1
The Real Numbers
1.1

Sets and Functions

Exercise 1.1.1. If a, b R and a < b, give a description in set theory notation for each of
the intervals (a, b), [a, b], [a, b), and (a, b] (see Example 1.1.1).
(a, b) = {x R : a < x < b}
[a, b] = {x R : a x b}
[a, b) = {x R : a x < b}
(a, b] = {x R : a < x b}.
Theorem 1.1.2. If A, B, and C are sets, then A (B C) = (A B) (A C).
Proof. If x A (B C), then x A and x (B C). Thus, either x B or x C. Thus,
x A B or in x A C. Thus, if x A (B C), then x (A B) (A C).
If an x (A B) (A C), then either x (A B) or x (A C). This means that surely
x A, and also that x B C. Hence, if x (A B) (A C), then x A (B C).
Therefore, A (B C) = (A B) (A C).
Exercise 1.1.3. Solved in Class
Question 1.1.5. What is the intersection of all the closed intervals containing the open
interval (0, 1)? Justify your answer.
Let A denote the
T set of all sets such that (0, 1) Ai . The intersection of all closed
intervals is denoted A. It is defined as
\
A = {x : x A A A}.
In other words, we are looking for some set A such that A is a subset of every other set in
A. This set is A = {x : 0 < x < 1}. Consider any other subset C of A. If C 6= A, then
necessarily there must exist an element x such that xT C and x
/ A, showing that A C
but C * A. Since x is not in every subset of A, x
/ A.
5

CHAPTER 1. THE REAL NUMBERS

Question 1.1.6. What is the union of all of the closed intervals contained in the open interval (0, 1)? Justify your answer.
Let A be the setScontaining all sets containing (0, 1)Sas a subset. The union of all these
sets is denoted by A. An object x is an element of A if there exists some set C A
such that x C. We need to consider two cases: either the object x 0 or 1 x. The
case of 0 < x < 1 is trivial. For any x such that 1 x we can create a set C such that
C = {y : 0 < y < x}.
S This since 1 x, it is guaranteed that C A. The case of x 0 is
analogous. Hence, A = (, ).
Problem 1.1.7. If A is a collection of subsets of a set X, formulate and prove a theorem
like Theorem 1.1.5 (from book numbering) for the intersection and union of A.
Theorem
1.1.7. Let A be a collection
A1 , A2 , ..., An of some set X. Then
S c
T c ofc subsets
c
c
c
c
( A) = A1 A2 ... An and ( A) = A1 A2 ... Acn .
Proof. ThisSis a generalization of DeMorgans law, proved
S in the book. We begin with the
statement ( A)c = Ac1 Ac2 ... Acn . We can rewrite ( A)c as (A1 A2 ... An )c . We
can then sub-partition this collection of unions into a collection of two unions, as such:
[
( A)c = [A1 (A2 ... An )]c
Then we will refer to A2 ... An as B. We can then rewrite the above as (A1 B)c , for
which DeMorgans laws apply. Thus, we write (A1 B)c = Ac1 B c = Ac1 (A2 ... An )c .
As next step, we sub partition B into two sets, as such
(A2 ... An )c = [A2 (A3 .... An )]c
Then DeMorgans laws apply again as above, and we can write [A2 (A3 .... An )]c =
Ac2 (A3 ... An )c . Since intersections and unions are associative, we can then write
[
( A)c = (Ac1 (Ac2 (A3 ... An )c )) = Ac1 Ac2 (A3 ... An )c
We continue
an inductive application of DeMorgans laws as outlined above, until we see
S
that ( A)c = Ac1 Ac2 ... Acn
The other proof is analogous, requiring a sub-partition of the collection of intersections and
rewriting them into series of intersections of two sets to which DeMorgans laws apply.
Problem 1.1.8. Which of the following functions f : R R are one to one and which ones
are onto. Justify your answer.
(a) f (x) = x2 ; This function is neither onto, nor one-to-one. It is not onto, since there is
no x such that f (x) < 0. It is not one-to-one since f (x) = f (x) for all x R.
(b) f (x) = x3 ; This function is both one-to-one and onto. It is one-to-one since there
f (x) 6= f (y) for all x, y such that x 6= y. It is onto, as for any y R, there exists an x R
such that f (x) = y.
(c) f (x) = ex This function is one-to-one, but not onto. It is one-to-one, for f (x) 6= f (y) for
all x, y R such that x 6= y. It fails to be onto since there exists no x such that f (x) < 0
for any x R.

1.1. SETS AND FUNCTIONS

Theorem 1.1.9. If f : A B is a function and E and F are subsets of B, then f 1 (E


F ) = f 1 (E) f 1 (F ).
Proof. If x f 1 (E F ), then f (x) E F . This means that f (x) is both in E as well as
in F . If f (x) E, then x f 1 (E). If f (x) F , then x f 1 (F ). Since f (x) is in both E
and F , x is in f 1 (E F ).
Assume x is in f 1 (E) f 1 (F ). Then, x f 1 (E) as well as x f 1 (F ). If x f 1 (E),
then f (x) E. If x f 1 (F ), then f (x) F . Since x is both in f 1 (E) as well as f 1 (F ),
we know that f (x) E F . This implies that x f 1 (E F ).
Since every x f 1 (E F ) implies that x f 1 (E) f 1 (F ) and vice versa, it is true that
f 1 (E F ) = f 1 (E) f 1 (F ).
Theorem 1.1.10. If f : A B is a function and E and F are subsets of B, then
f 1 (E\F ) = f 1 (E)\f 1 (F ) if F E.
Proof. If x f 1 (E\F ), then f (x) E\F . Thus f (x) E but f (x)
/ F . This means that
1
1
1
1
x f (E) and but also x
/ f (F ). In other words, x f (E)\f (F ).
1
Assume now that x f (E)\f 1 (F ). Then x f 1 (E) but x
/ f 1 (F ). This means that
1
f (x) E\F , and hence x f (E\F ).
It follows that f 1 (E)\f 1 (F ) = f 1 (E\F ).
Theorem 1.1.11. If f : A B is a function and E and F are subsets of A, then f (EF ) =
f (E) f (F ).
Proof. If y f (E F ), then y = f (x) for some x E or x F . If x E, then y f (E).
If x F , then y f (F ). Since x is in either one of these, we know that y f (E) f (F ).
Assume now that y f (E) f (F ). This implies that y = f (x) for some x E or x F .
Thus we can write x E F . Then y f (E F ).
Since any element of f (E F ) is in f (E) f (F ) and vice versa, we conclude that f (E F ) =
f (E) f (F ).
Theorem 1.1.12. If f : A B is a function and E and F are subsets of A, then f (EF )
f (E) f (F ).
Proof. Assume that y f (E F ). Then y = f (x) for some x E F . This means
that both x E as well as x F . Then, f (x) f (E) and f (x) f (F ), showing that
f (x) f (E) f (F ), or - equivalently - that y f (E) f (F ). This proves that f (E F )
f (E) f (F ).
Question 1.1.13. Give an example of a function f : A B and subsets F E of A for
which f (E)\f (F ) = f (E\F ).
The above conditions are fulfilled for a function f (x) = x with A = B = [0, 10], and the
subsets E = [1, 6] and F = [1, 2] E.
Exercise 1.1.14. Solved in Class
Exercise 1.1.15. Solved in Class

CHAPTER 1. THE REAL NUMBERS

1.2

The Natural Numbers

1.3

Integers and Rational Numbers

Exercise 1.3.1. Given that N has an operation of addition which is commutative and associative, how would you define such an addition operation in Z?
Assume a, b Z. We differentiate three cases:
a) Both a, b N. In this case, proceed as in N.
b) Both a, b < 0. In this case a + b = (c + d) where c = a and d = b. As such, we may
proceed as in the case of natural numbers.
c) assume a > 0, b < 0. We now distinguish two cases. Provided a > b, we may reduce a
to a sum of two integers m, n N such that n = b. We then define a + b = m + n + b = m,
since n + b = 0 by definition of the additive inverse. Assume now instead that a < b. In
this case, we can split b into an addition problem, with two integers m, n N such that
m + n = b and a = m. Then
a + b = a + ((m + n)) = a + (m) + (n) = n.
This defines addition in Z.

1.4

The Real Numbers

Exercise 1.4.1. For each of the following sets, describe the set of all upper bounds for the
set:
(a) the set of odd integers; The integers are unbounded.
(b) {1 1/n : n N}; The set of all upper bounds for this set is {x N : x 1}.
(c) {r Q : r3 < 8}; The set of all upper bounds for this set is {x Q : x 2}.
(d) {sin x : x R}; The set of all upper bounds for this set is {x R : x 1}.
Exercise 1.4.2. For each of the sets in (a), (b), (c) of the preceding exercise, find the least
upper bound of the set, if it exists.
(a) There is no upper bound, and hence no least upper bound.
(b) The least upper bound is 1.
(c) The least upper bound is 2.
Theorem 1.4.3. If a subset A of R is bounded above, then the set of all upper bounds for
A is a set of the form [x, ). What is x?
Proof. Let B denote the set of all upper bounds of A. By definition, a number m R is
considered an upper bound for the set A if z m for all z A. If the set A has a largest
number, then this largest number - y 0 - will be in the set B. In that case, it is obvious that
all numbers m > y 0 will also be upper bounds, since we assumed that x y 0 for all x A,
and that m > y 0 , it follows that x y 0 < m. Therefore, the set [y 0 , ) would be the set of

1.5. SUP AND INF

all upper bounds of A.


Assume now that A does not have a largest number. By the completeness theorem we know
that any subset A of an ordered field - such as R - is indeed bounded above. Specifically,
according to theorem 1.4.4 of the book we know that any subset of R not only is bounded
above, but has a least upper bound. By definition, a number c is a least upper bound if and
only if it is a number such that x c for all x A, and for every k R, if k is an upper
bound of A, then k c. It is obvious then that the set of all upper bounds of A will be the
set [c, ) where c is the least upper bound of A.
Exercise 1.4.4. If x2 < 1 x, then x2 + x 1 < 0, hence
!
!
1 + 5
1 5
x
x
< 0,
2
2

1 5 1 + 5
1 + 5
so x
,
, hence A is bounded above by
, which is its least
2
2
2
upper bound.
Exercise 1.4.6. For the forward direction, observe that for each pair x, y F with x > 0,
y/x in F and since F is archimedean, we have that for some integral n, n > y/x so nx > y
as desired.
For the reverse direction, we have for each pair x, y F, x > 0, there exists integral n
with nx > y. In particular, let x = 1 > 0, so n > y. Hence F is archimedean.
Exercise 1.4.7. By the archimedean property, there exists natural n with y x > 1/n.
Let k be an integer with k/n < x < (k + 1)/n, then adding the inequalities y x > 1/n and
x > k/n gives us y > (k + 1)/n, so (k + 1)/n is a rational between x and y.
This proves the claim if both x, y are positive. If both are negative, the proof works
essentially the same way. If one is positive and the other negative, then take 0 as the
rational number between the two.
Exercise 1.4.8. . Suppose not, so that x + r = s Q and xr = s0 Q, then x = s r Q
and x = s0 /r Q, both of which lead us to a contradiction.

1.5

Sup and Inf

Exercise 1.5.1. For each of the following sets, find the set of all extended real numbers x
that are greater than or equal to every element of the set. Then find the sup of the set. Does
the set have a maximum?
(a) (10, 10); The set of all numbers greater than this set is the set [10, +). The supremum
of the set in question is 10. The set does not have a maximum.
(b) {n2 : n N}; In the extended set of real numbers, the only element greater than or equal
to all the elements in the set in question is +, which thereby must also be its supremum.
The set does not have a maximum.
}; The set of all real numbers greater than the set in question is the set [2, ). The
(c) { 2n+1
n+1
supremum is 2 and the set does not have a maximum.

10

CHAPTER 1. THE REAL NUMBERS

Exercise 1.5.2. Find the sup and inf of the following sets. Tell whether each set has a
maximum or a minimum.
(a) (2, 8]; The infimum of the set is 2 and the supremum is 8. The has a maximum, but
not a minimum.
(b) nn+2
2 +1 ; The infimum of the set is 0, and the supremum is 2. The set has a maximum, but
no minimum.

2
2
(c) {n/m : n,
m Z, n < 5m }; The infimum of the set is 5, and the maximum is 5.
Seeing that 5 is not a rational number, the set has neither a maximum nor a minimum.
Exercise 1.5.3. Prove that if sup A < , then for each n N there is an element an A
such that sup A 1/n < an sup A.
Proof. This is true since we can easily construct an element an such that this equality holds.
We assume that A is defined for all m/n with m, n Z within A. In this case, we constructs
our term to be an = sup A 1/(n + 1). It is obvious that since 1/(n + 1) < 1/n, that
sup A 1/n < sup A 1/(n + 1) sup A.
Alternatively, we may also note that sup A 1/n < sup A for all n N by definition, so the
inequality holds in the trivial case of an = sup A.
Exercise 1.5.4. Prove that if sup A = , then for each n N there is an element an A
such that an > n.
Proof. Assume some set A whose supremum is +. In that case, x A, x < . Both
from the Archimedean property and from the Peano Axioms we know that for every n N,
there is a successor element n0 which is also in N, such that n < n0 . Since there @a such
that a = , and n < , this implies that an such that an = n0 and an A, showing that
n < an < .
Exercise 1.5.5. Formulate and prove the analog of Theorem 1.5.4 for inf.
Theorem. Let A be a non-empty subset of R and let x be an element of R. Then
(a) inf A x if and only if a x for every a A;
(b) x > inf A if and only if x > a for some some a A.
Proof. By definition, a x if and only if x is a lower bound for A. If x is a lower bound for
A, then A is bounded below. This implies that its inf is its greatest lower bound, which is
necessarily greater than or equal to x. Conversely, if inf A x, then inf A is finite and is the
greatest lower bound for A. Since inf A x, x is also a lower bound for A. Thus, inf A x
if and only if a x for every a A.
If x > inf A, then x is not a lower bound for A, which means x > a for some a A.
Conversely, if x > a for some a A, then x > inf A, since a inf A. Thus, x > inf A if and
only if x > a for some a A.
Exercise 1.5.6. Prove part (d) of Theorem 1.5.7.
Theorem. Let A, B be non-empty subsets of R. Then sup(A B) = sup A inf B.

1.5. SUP AND INF

11

Proof. According to the book, sup(A + B) = sup A + sup B (proof on p. 30). We can
then write sup(A + (B)) = sup A + sup(B). We then apply Theorem 1.5.7b, to rewrite
sup(B) as inf A. From this it follows that
sup(A + (B)) = sup(A B) = sup A + ( inf B) = sup A inf B
.
Exercise 1.5.7. Prove (e) of Theorem 1.5.7.
Theorem. Let A, B be non-empty subsets of R. If A B, then sup A sup B and
inf B inf A.
Proof. If A B, then a A implies that a B for all a. Then, if sup A A, sup A B.
Since sup B b for all b B, it is obvious that sup A sup B. Assume now that sup A
/ A.
In that case, sup A  A for all  > 0. Thus, sup A  A and sup A  B. By
definition, sup B is greater than or equal to all b B. This means that if sup A  B
implies that sup A sup B. The proof for the infimum is analogous.
Exercise 1.5.10. Prove (a) of Theorem 1.5.10.
Theorem. Let f and g be functions defined on a set containing A as a subset, and let
c R be a positive constant. Then supA cf = c supA f and inf A cf = c inf A f .
Proof. Let f be function f : A B. Then sup f is the supremum of B provided that f is
surjective. Let M be an arbitrary upper bound of cx for some x B. We say that cx M
if and only if x M/c. This shows that M is an upper bound of cx if and only if M/c is an
upper bound of B. Hence, sup cB = c sup B and similarily sup cf = c sup f . The result for
the infimum follows similarily.
Exercise 1.5.8. [(a)]
supI f = 1, inf I f = 0. These are the max and min respectively.
2.
1. supI f = , inf I f = 3 at x = 2. Neither are the max nor min of f on I.
3. inf I f = 0, supI f = 1. The inf of f on I is actually a minimum, but the sup is not a
maximum.
Exercise 1.5.9. Solved in Class
Exercise 1.5.11. Prove (b) of Theorem 1.5.10.
Theorem. Let f and g be functions defined on a set containing A as a subset, and let
c R be a positive constant. Then supA (f ) = inf A f .

12

CHAPTER 1. THE REAL NUMBERS

Proof. We have a function f : A B. A number x is a lower bound for f (a) for all
a A if and only if x is an upper bound for the set f (a). Let L be the set of all lower
bounds for f (a). Then L is the set of all upper bounds for f (a). Furthermore, the
largest member of L and the smallest member of L are negatives of each other. That is,
inf f (a) = sup(f (a)), or equivalently inf f = sup(f ).
Exercise 1.5.12. Prove (c) of Theorem 1.5.10.
Theorem. Let f and g be functions defined on a set containing A as a subset, and let
c R be a positive constant. Then supA (f + g) supA f + supA g and inf A f + inf A g
inf A (f + g).
Proof. By definition, f (a) sup f for all a A and g(a) sup g for all a A. Therefore,
f (a) + g(a) sup f + sup g. Let c denote the supremum of f + g. We know that sup f + sup g
is an upper bound for f (a) + g(a). Since the supremum is always less than or equal to an
upper bound, we find that c sup f +sup g. This implies that sup(f +g) sup f +sup g.
Exercise 1.5.13. Prove (d) of Theorem 1.5.10.
Theorem. Let f and g be functions defined on a set containing A as a subset, and let
c R be a positive constant. Then sup{f (x) f (y) : x, y A} = supA f inf A f .
Proof. This appears somewhat obvious. The function f is defined on A, i.e., for every a A,
f maps to some value f (a) in some set, lets call it B. The value sup f is defined as to be
the least upper bound of f (a), i.e. @x such that f (x) > sup f for some x A. The infimum
is defined as the value such that there is no value x A such that x < inf f . The value
defined by f (x) f (y) for all x, y A is a measure of the distance between these two values.
Since sup f and inf f are defined as above, we can see that there cannot be a greater distance
between any other two points in B than the distance between sup f and inf f . Therefore,
for any collection of distances between points in B reached by f (x) for all points x A, the
supremum of this collection - namely, the largest value of this set such that no other value
is larger - cannot be any other than the distance between the supremum and the infimum of
the function itself.

Chapter 2
Sequences
2.1

Limits of Sequences

Exercise 2.1.1. Show that


(a) if |x 5| < 1, then x is a number greater than 4 and less than 6.; This is
equivalent to saying 1 < x 5 < 1. We add 5 to the inequality, and we get 4 < x < 6.
(b) if |x 3| < 1/2 and |y 3| < 1/2, then |x y| < 1; We add the inequalities, such that
we see |x 3| + |y 3| < 1/2 + 1/2 = 1. We notice that |y 3| = |3 y|. We rewrite using
the triangle inequality:
|(x 3) + (3 y)| |x 3| + |3 y| < 1
|x y| |x 3| + |3 y| < 1.
(c) if |x a| < 1/2 and |y b| < 1/2, then |x + y (a + b)| < 1. We add the inequalities
and get |x a| + |y b| < 1/2 + 1/2 = 1. We can then rewrite using the triangle inequality
as above
|(x a) + (y b)| |x a| + |y b|
|x + y a b| |x a| + |y b|
|x + y (a b)| |x a| + |y b|

<1
<1
< 1.

Exercise 2.1.2. Adding the two inequalities gives


|x 1| + |x 2| < 1,
hence by the triangle inequality |2x 3| |x 1| + |x 2| < 1, so |x 23 | < 21 . Hence
x (1, 2). Combining this with |x 1| < 1/2 and |x 2| < 1/2 we have x (1, 23 ) and
x ( 32 , 2), a contradiction.
Exercise 2.1.3. Put each of the following sequences in the form a1 , a2 , a3 , . . . , an . This
requires that you compute the first 3 terms and find an expression for the nth term.
(a) the sequence of positive odd integers; This is a sequence of the form 1, 3, 5, . . .. To
find the n th term, we express this sequence as an = 2n 1, with n N.
(b) the sequence defined inductively by a1 = 1 and an+1 = a2n ; The sequence begins
13

14

CHAPTER 2. SEQUENCES

with 1, 1/2, 1/4, . . .. The nth term will be something like an = ((1)n1 )/(2n1 ) for n N.
an
. This is the series
(c) the sequence defined inductively by a1 = 1 and an+1 = n+1
2
1, 1/3, 1/12, 1/60, . . .. The nth term is: an = (n+1)! .
Exercise 2.1.4. Find lim 1/n2 .
The larger n become, the smaller 1/n2 will become. We guess the limit to be 0. For
2
any  > 0, we need an N such that whenever n >
p N , 1/n < . We find that this is true
2
whenever 1/ < n , or in other words - whenever 1/ < n.
2n1
.
Exercise 2.1.5. Find lim 3n+1

We guess the limit to be 2/3.


|

2n 1 2
3(2n 1) 2(3n + 1)
6n 3 6n 2
|=|
|=|
|
3n + 1 3
3(3n + 1)
9n + 3
5
5
5
=|
<| |<| |
9n + 3
9n
n

We must choose an n > N such that N >

5


so that this will be true.

Exercise 2.1.6. Find lim(1)n /n


n

We guess the limit to be 0. We see | (1)


| = | n1 |. Hence we need to choose an n > N
n
such that N > 1 for this inequality to be true.
1
Exercise 2.1.7. The limit is zero. Let N = . Then if n > N ,



n

n
n
1
1


n3 + 4 0 = n3 + 4 < n3 = n2 < N 2 = .
Hence lim

n n3

n
= 0.
+4



1 2 2
. Then for n > N ,
Exercise 2.1.8. The limit is zero. Let N =
2


1 2

2 n > |1 2 |,
n >

2
hence adding 2 + n to both sides yields

2 + n + 2 n > |1 2 | + 2 + n 1 + n,
where the last step follows from the triangle inequality. Factoring the left side gives

( n + )2 > n + 1,
and since both sides are positive we may take the square root which yields

n +  > n + 1  > n + 1 n = | n + 1 n 0|,

hence lim n + 1 n = 0.
n

2.2. USING THE DEFINITION OF LIMIT

15

Exercise 2.1.9. The limit is zero. We know for any  > 0, there exists N1 such that for all
n > N,
1
< /2,
n
since /2 is another positive real and we have shown the limit of 1/n as n goes to infinity is
zero in problem 4. Similarly, we may find N2 such that for all n > N2 ,


(1)n


n2 < /2,
since we found the limit as n of (1)n /n earlier to be 0, and it can be shown in much
the same way that lim (1)n /n2 = 0. Hence, for n > max{N1 , N2 }, both inequalities hold
n
and we may add the two and apply the triangle inequalityto obtain



1 (1)n
1 (1)n
+
0 + 2 < ,
n
n2
n
n
hence the limit is zero.
Exercise 2.1.10. Prove that lim 2n = 0. Hint: prove first that 2n n for all natural
numbers n.
Proof. We wish to show that 2n > n for all n. Proof by induction. The base case, 21 > 1
is obviously true, since 21 = 2. We assume now that 2n > n for some n. Then we wish to
check 2n+1 . But, we can rewrite this simply as 2n 21 . Let k = 2n . Since we know that k > n,
it is obvious that 2k > n + 1. Thus, 2n > n for all n N.
We note that 2n = 21n . Thus, lim 2n = lim 21n . Since 2n increases until infinity, we see that
1/2n will grow smaller and smaller, since 1/2n > 1/2n+1 for all n.
We see that for any  > 0, we need to simply pick n such that 1/ < 2n . As such, the limit
is 0.
Exercise 2.1.11. Prove that if an 0 and k is any constant, then kan 0.
If an 0, this means that an <  for any  > 0. We multiply by k and find that kan < k.

2.2

Using the Definition of Limit

Exercise 2.2.1. Make an educated guess as to what you think the limit is, then use the
definition of limit to prove that your guess is correct.
2

2
2
= 3. Hence, the limit
lim 3n
. I assume the limit will be 3. We note that 3n
< 3n
n2 +1
n2 +1
n2
is 3.
1
Exercise 2.2.2. The limit is zero. Let N = , then



n


= n < n = 1 < 1 = .

0
n2 + 2
n2 + 2
n2
n
N

16

CHAPTER 2. SEQUENCES

Exercise 2.2.3. Make an educated guess as to what you think the limit is, then use the
definition of limit to prove that your guess is correct.
1
|; therefore, this is true whenever
lim 1n I assume the limit will be 0. We see | 1n | = | n1/2

we choose an n > N such that N > 1 .

2
Exercise 2.2.4. The limit is 1. Let N = , then



2

2n 1
n


= 2n + 1 < 2 .
1 = 2

n+1

n + 2n + 1 n2 + 2n + 1
n
This last step is true because for any positive integer n, we have 3n + 2 > 0, hence 2n2 + 4n +
2 > 2n2 + n, and so dividing by n(n2 + 2n + 1) on both sides yields the desired inequality.
Hence,


2
2

2
n


= .
1 < <

n
n+1
N
Exercise 2.2.5. Make an educated guess as to what you think the limit is, then use the
definition of limit to prove that your guess is correct.
1
1
. Let A = {n N : |an | } for a given . This set is finite since
2
2

2

1
1
2
2
2
n +nn n +nn + +
+ n(2 + 1)
2
2

2
1
0 +
+ 2n.
2

The limit is

But this last inequality is never true, since each of the terms is strictly greater than zero. Of
course, we might also have
1
n + n2 + n  n2 + +n(1 2) + 2 n2 + n
2
2n + 2 0

n,
2
which is certainly only true for finite n. Hence for a given  the set A is in fact finite, so the
1
limit is .
2
Exercise 2.2.6. Make an educated guess as to what you think the limit is, then use the
definition of limit to prove that your guess is correct.
lim(1 + 1/n)3 = 1. Proof:
|(1 +

1 3
1
3
3
1
3
3
) 1| = | 3 + 2 + + 1 1| = | 3 + 2 + |
n
n
n
n
n
n
n

2.2. USING THE DEFINITION OF LIMIT

17

We note that each term is of the form c/n or multiples thereof for some constant c. It has
already been shown that each such term tends can be made smaller than any . This also
holds for the sum.
Exercise 2.2.7. Suppose the sequence converges to a, then for all  > 0, there is an N
so that when n > N , |an a| <  a  < an < a + . In particular, let  = 1, then
certainly for all n > N , the sequence is bounded above by a + 1 and below by a 1. For
n < N, notice that the a1 , ..., aN is bounded above by e = max{|a1 |, ..., |an |} and below
by f = min{a1 , ..., aN }. Hence the entire sequence is bounded above by max{e, a + 1} and
min{f, a 1}. 
Exercise 2.2.8. Prove that if lim an = a, then lim a3n = a3 .
|a3n a3 | = |(an a)(a2n + an a + a2 )|
We then note that we are given that |an a| < . From this we see that
|(an a)(a2n + an a + a2 )| < (a2n + an a + a2 ).
Exercise 2.2.9. Does the sequence {cos(n/3)} have a limit? Justify your answer.
No. The sequence {cos(n/3)} oscillates between 1 and 1; a limit cannot converge to
two different values. Hence, this sequence does not have a limit.
Exercise 2.2.10. For a similar reason as in the previous problem, the sequence an =
cos(n) does not converge but the sequence |an | = | cos(n)| does converge since |an | = 1
for all n.
Exercise 2.2.11. Prove that if {an } and {bn } are sequences with |an | bn for all n and if
lim bn = 0, then lim an = 0 also.
We are given that |an | bn for all n. Therefore, we know that lim |an | lim bn . We
know that lim bn = 0. Hence we can write - equivalently - that lim |an | 0. We notice that
|an | is defined to be greater than or equal to zero. Hence we have 0 lim |an | 0, from
which it follows by the squeeze theorem (proof on p. 43 of the book) lim |an | = 0.
Exercise 2.2.12. Prove the following partial converse to Theorem 2.2.3: Suppose {an } is a
convergent sequence. If there is an N such that an c for all n > N , then lim an c. Also,
if there is an N such that b an for all n > N , then b lim an .
Note that an is bounded by c according to the premise. In this case, we can say that
an sup an c for all n. Let a = lim an . We know by definition that a sup an , and
therefore we can write that lim an sup an c.
Likewise, we can say that b is a lower bound for an such that b inf an . We know that by
definition inf an a, allowing us to write b lim an .

18

CHAPTER 2. SEQUENCES

Exercise 2.2.15. Suppose this sequence converges to


zero. 
Thenfor all  > 0, there exists
N
+ 1 106 . This is a multiple
N such that for all n > N , |an | < . But consider n =
6
10
of one million, so an . And, it is larger than N since


N
N
+1
<
106
106



N
6
by definition, 10
+ 1 > N. Hence we have a contradiction.
106

2.3

Limit Theorems

Exercise 2.3.1. Solved in Class


2

n 5
Exercise 2.3.2. Use the Main Limit Theorem to find lim n3 +2n
2 +5 .

n2 5
=
n3 + 2n2 + 5
1/n 5/n3
lim
=
1 + 2/n + 5/n3
lim(1/n 5/n3 )
0
= = 0.
3
lim(1 + 2/n + 5/n )
1
lim

(dividing top and bottom by n3 )

Exercise 2.3.3. By dividing numerator and denominator by 2n , in a similar fashion to


problem 1, we can see that the limit is 1.
Exercise 2.3.4. Let an = 1/n, then lim an = 0, and let bn = sin n, then |bn | 1. By
sin n
= an bn has limit zero.
theorem 2.3.2, the sequence
n
Exercise 2.3.5. Prove Theorem 2.3.2.
Proof. We know that lim an = 0, hence we know that for all  > 0, there exists an N such
that whenever n > N , |an | < . Likewise, we know that bn is bounded, such that we can

state that q bb q. We can then also write |an | < |q|
. We guess that the limit of (an )(bn )
is zero, so we write:
|an bn 0| = |an bn | |an q|

|an bn | |q|
|q|
|an bn | .
Thus, the lim an bn = 0, since we there is an N such that the above inequality is true whenever
we pick an n > N .

2.3. LIMIT THEOREMS

19

Exercise 2.3.6. Prove that a sequence {an } is both bounded above and bounded below if
and only if its sequence of absolute values {|an |} is bounded above.
Proof. By definition, if {|an |} is bounded above, then there exists some M such that |an | M
for all n. This is equivalent to saying M an M , which proves that {an } is bounded
above and below.
Exercise 2.3.7. Prove part(b) of Theorem 2.3.6.
Proof. Since both an and bn have a limit, we can write |an a| < 2 and |bn b| < 2 . For
all , we have an N such that if we choose n > N , these inequalities are true. We know add
them together and find
|an a| + |bn b| < 
|(an ) + (bn b)| |an a| + |bn b| < 
|(an + bn ) (a + b)| |an a| + |bn b| < .

Exercise 2.3.8. Prove that if {bn } is a sequence of positive terms and bn b > 0, then
there is a number m > 0 such that bn m for all n.
This is true by virtue of the definition of R. The statement above is equivalent to saying
that we are looking for some m such that 0 < m bn . By definition R is full, such that
between any two numbers, there are infinitely more numbers.
Exercise 2.3.9. Prove part (d) of Theorem 2.3.6. Hint: Use the previous exercise. I.e, that
if an a and bn b, an /bn a/b, if b 6= 0 and bn 6= 0 for all n.
Proof.
|an

1
1
1
1
1
1
1
1
|
a | = |an a + a a | |an = a|| | + |a||
bn
b
bn
bn
bn
b
bn
bn b

We know that {1/bn } is bounded, and hence {|1/bn |} is bounded above. We also have
|an a| 0. Therefore, |an a||1/bn | 0. Also, |a||1/bn 1/b| 0. By (b) we know that
|an a||1/bn | + |a||1/bn 1/b| 0, proving that an /bn a/b.
Exercise 2.3.10. Prove part (f) of theorem 2.3.6. Hint: use the identity:
xk y k = (x y)(xk1 + xk2 y + . . . + y k1 )
1/k

with x = an

1/k

and y = a1/k , to show that an a1/k if an 0 for all n.

Proof. We notice that


1/K
+ a1/K2
a1/k + . . . + a1/K1 ) = (an a)bn
a1/k
= (an a)(a1/K1
n
n
n a

20

CHAPTER 2. SEQUENCES

where
bn = a1/K1
+ a1/K2
a1/k + . . . + a1/K1
n
n
We know that {an } converges, and hence that {|an |} is bounded above. We choose an upper
1/k

sowing that {|bn |} is


bound m for {|an |} which satisfies that |an | m. Then bn 1/k
m
bounded above. According to theorem 2.3.2 we conclude that |an a||bn | and find from
1/k
theorem 2.3.1 that an a1/k .
Exercise 2.3.12. Prove that lim a1/n = 1. Hint: use the result of the previous exercise.
We notice that n1/n > 1 for all n N.We can therefore write that we are looking for a
solution to n1/n 1 < .We can rearrange and raise both sides to the nth power, resulting in
the equation n < (1 + )n . We can expand the right hand side using the binomial theorem:
1
n < 1 + n + n(n 1)2 + . . .
2
As long as n < 21 n(n 1)2 this inequality holds, requiring that n > 1 +
any  > 0 there exists an N such that whenever n > N , |n1/n 1| < .

2.4

2
.
2

Therefore, for

Monotone Sequences

Exercise 2.4.1. Tell which of the following sequences are non-increasing, non-decreasing,
bounded? Justify your answers.
(a) {n2 }; for n N, this sequence is non-decreasing since n2 < (n + 1)2 for all n. It is
bounded below by 1.

(b) { 1n }; this sequence is non-increasing, since n = n1/2 < (n + 1)1/2 for all n. This then
1
implies 1n > n+1
. The sequence is bounded by 0 and 1.
n

(c) { (1)
}; this sequence is neither non-increasing, nor non-decreasing as the sign of the
n
value of the sequence fluctuates due to the term (1)n . It is, however, bounded by 1 and
1/2.
(d) { 2nn }; this is the sequence 12 , 24 , 38 , . . . which is clearly non-increasing. It is bounded by 0
and 1.
n
(e) { n+1
}; this is the sequence 12 , 23 , 43 , 45 , . . . which is clearly non-decreasing and tending to
1. It is bounded by 1/2 and 1.

Exercise 2.4.2. Prove that the sequence xn with x1 = 1 and xn+1 = xn + 1 converges
and decide what number it converges to.
q
p
p

Proof. The first few terms of the sequence: 1, 2, 1 + 2, 1 + 1 + 2, . . .. We see


that xn is both increasing an bounded:
s
s
r
r
q
q

xn+1 = 1 + 1 + . . . 1 + 2 > 1 + 1 + . . . 1 + 2 = xn
|
{z
} |
{z
}
n terms

n 1 terms

2.4. MONOTONE SEQUENCES

21

Then xn is increasing for


all n N. We
xn < 2 by induction. We have x1 = 1 < 2.
prove that
Then, if xk < 2, xk+1 = 1 + xk < 1 + 2 = 3 < 2. Thus by the monotone convergence
theorem, xn converges and is bounded
by 2.

Finding thelimit: We solve


a
=
1
+
a,
which implies a2 a 1 = 0. The solutions to this

are a1 = 1+2 5 , a2 = 12 5 . The correct limit is a1 , since a2 < 0.


Exercise 2.4.3. If a1 = 1 and an+1 = (1 2n )an , prove that {an } converges.
Proof. We notice that 2n is monotone and converges to 0. Therefore we see that 1 2n is
also monotone, converging to 1. The whole term then is monotone and non-increasing. It is
also bounded by 0 and 1. Therefore, by the monotone convergence theorem, an converges.
Exercise 2.4.4.
5

+2
Exercise 2.4.8. Prove that lim nn+3n
4 n+1 = .

lim

n5 + 3n3 + 2
n5 (1 + 3n3 /n5 + 2/n5 )
=
lim
n4 n + 1
n4 (1 n/n4 + 1/n4 )
1 + 3/n2 + 2/n5
= lim |{z}
n (
)
1 1/n3 + 1/n4
|
{z
}
1

+2
And hence, lim nn+3n
4 n+1 = .

Exercise 2.4.10. If the sequence is bounded, then we are done by theorem 2.4.1. Otherwise
there is no M R such that M an for all n. This is equivalent to saying any M R, there
exists N such that aN > M . Since the sequence is monotone, for all n > N , an > aN > M .
Hence it has limit equal to infinity.
Exercise 2.4.11. Prove Part (c) of Theorem 2.4.7.
Theorem lim an = iff lim(an ) =
Proof. If an , there exists some value of an such that an > M for any possible M R.
If we consider the sequence an (1), we see clearly that an < M for any M R. But if
that is true, then lim an = .
Exercise 2.4.12. Suppose lim bn < . Then bn is bounded by theorem 2.2.3, so there exists
M with bn M for all n. Since an bn , we have an M so lim an < , a contradiction.
Exercise 2.4.13. For all M R, given k 0, M/k R. Hence there exists N , for all
n > N , an > M/k. Since bn k, we have an bn > k M/k = M, hence lim an bn = .
Exercise 2.4.14. Solved in Class

22

CHAPTER 2. SEQUENCES

2.5

Cauchy Sequences

Exercise 2.5.1. Give an example of a nested sequence of bounded open intervals that does
not have a point in its intersection.
Exercise 2.5.2. Let
\ In = [n, ) which are nested, closed and unbounded. Suppose there is
an x R with x
Ii . But there exists an integer m, m > x by the archimedean property.
i

Hence Im does not contain x, a contradiction.


Exercise 2.5.4. Prove by induction that if {nk } is an increasing sequence of natural numbers, then nk k for all k.
Proof. Assume the base case nk = n, which is the series 1, 2, 3, 4, 5, . . .. Since k is the counter
index, i.e. k N, it is obvious that nk = k = 1, 2, 3, 4, 5, . . .. We generalize to the n + 1 case,
i.e. nk = n + 1. In that case we have nk = n + 1 = k + 1 > k.
Exercise 2.5.5. Which of the following sequences {an } have a convergent subsequence?
Justify your answer.
(a) an = (2)n ; None of the subsequences are convergent, as they either tend to + or .
nn
; This sequence is convergent for all n such that n mod 2 = 0, which is
(b) an = 5+(1)
2+3n
the sequence starting with 0.875, 0.6428, 0.55, 0.5, 0.46875, 0.4473, 0.4318, 0.42, 0.41071, . . .
n
(c) an = 2(1) This sequence has convergent subsequences for all n such that n mod 2 = 0
and for n mod 2 = 1.
Exercise 2.5.6. [(a)]
The sequence is 1, 1, 1, 1, 1, 1, .... The subsequence {ani }, ni = 2i 1 converges to
1.

2.
1. The sequence is 2/2, 1, 2/2, 2/2, 1, 2/2, 0, .... Since the sequence is periodic, we can find infinitely many terms equal to 0, which gives a convergent subsequence
(convergent to 0).
3. Let ank = 1/2k which happens when n = 2k + 1. Then by a previous homework this
has limit 0. Hence it is a convergent subsequence.
Exercise 2.5.7. For each of the following sequences, determine how many different limits
of subsequences there are. Justify your answer.
(a) {1 + (1)n }; This sequence is 0, 2, 0, 2, 0, 2, . . . and as such has two different limits: 0
and 2.
(b) {cos(n/3)}; There are two different limits. The first approaches 1 for the sequence of all
n where n mod 6 = 0. The second limit is attained at 1 for all n such that n mod 6 = 3.
(c) 1, 12 , 1, 21 , 13 , 1, 21 , 31 , 14 , 1. 12 , 13 , 41 , 15 , . . . The terms a1 , a3 , a6 , a10 , a1 5, . . . are convergent to 1.
Exercise 2.5.8. Does the sequence sin n have a convergent subsequence? Why?
Yes, it has three convergent subsequences, provided n R. If n N, then it is not
convergent.

2.6. LIM INF AND LIM SUP

23

Exercise 2.5.9. Prove that a sequence which satisfies |an+1 an | < 2n for all n is a Cauchy
sequence.
Proof. We notice that the sequences defined by the above condition are non-increasing and
covergent. We notice the following pattern:
|an+2 an | = |an+2 an+1 + an+1 an | |an+2 an+1 | + |an+1 an | < 2n+1 + 2n
|an+3 an | = |an+3 an+2 + an+2 an+1 + an+1 an | |an+3 an+2 | + |an+2 an+1 | + |an+1 an |
< 2(n+2) + 2(n+1) + 2n
Inductively, we see that this pattern continues for all patterns an and an + k with k N.
Now, we assume two indices m and n such that m > n. We find
|am an | < 2(m1) + 2(m2) + . . . + 2n =
2n (1 + 21 + 22 + . . . + 2(m1)+n )
{z
}
|
geometric series

We rewrite and solve the geometric series:


2n (

0
X

2k ) = 2n (2 2m+n+1 ) = 21n 21m

k=m+n+1

We want to to prove |21n 21m | 21n + 21m < . We solve the equations 21n <
log()
21m < 2 . The solution to this is 2 log(2)
< n, m


2

and

Exercise 2.5.10. Solved in class

2.6

lim inf and lim sup

Exercise 2.6.1. Solved in Class


Exercise 2.6.2. Find lim inf and lim sup for the sequence an =
largest integer k so that 2k n.

n
2kn

1 with kn being the

This is the sequence 0, 0, 12 , 0, 14 , 24 , 43 , 0, 18 , 28 , 83 , 48 , 58 , 68 , 78 , 0, . . .. It is clear that lim inf = 0


and lim sup = 1.
Exercise 2.6.3. Find lim inf and lim sup for the sequence 1, 21 , 1, 12 , 13 , 1, 21 , 13 , 14 , 1, . . ..
We find that lim inf = 0 and lim sup = 1.
Exercise 2.6.4. Solved in class
Exercise 2.6.5. If lim sup an is finite, prove that lim inf(an ) = lim sup an .

24

CHAPTER 2. SEQUENCES

Proof. By assumption, lim sup an is equal to some a, such that a an for all an {an }.
We multiply this by 1 to find the inverse sequence {an }. Then we have a an for
all an {an }. By definition, this means a = lim inf(an ). Therefore, lim inf(an ) =
lim sup an .
Exercise 2.6.7. Solved in Class
Exercise 2.6.8. If {an } and {bn } are non-negative sequences and {bn } converges, prove
that lim sup an bn = (lim sup an )(lim bn ).
Proof. We need to consider two cases. First, assume {an } is not bounded above. Then
lim sup an = . It then doesnt matter what we multply an with, we will always get infinity
provided that bn 6= 0.Then lim sup(an bn ) = lim sup an lim bn = .
We now consider case 2, where an is bounded above. By Bolzano-Weierstrass we know that
an then has at least one convergent subsequence. Let a be the subsequential limit of an , and
let M be the upper bound of an . We know then that lim sup an exists and lim sup an M .
MORE WORK NEEDED ON THIS. We note that according to the main limit theorem, if
an a and bn b, an bn ab. Thus lim sup(an bn ) = lim sup an lim bn .
Exercise 2.6.9. Solved in class
Exercise 2.6.12. Which numbers do you think are subsequential limits of {sin n}
n=1 ? Can
you prove that your guess is correct?
All x R with |x| 1 are limits for sin.

Chapter 3
Continuous Functions
3.1

Continuity

Exercise 3.1.1. If f is a function with domain [0, 1], what is the domain of f (x2 1)?
======= If f is a function with domain [0, 1], what is the domain of f (x2 1)?
g is defined at point x iff x2 1 [0, 1], 0 x2 1 1.
(
x2 1 x (, 1] [1, ]

x2 2 x [ 2, 2]

Thus x [ 2, 1] [1, 2].


Exercise 3.1.2. What is the natural domain of the function
is this function continuous? Why?

x2 +1
?
x2 1

With this as its domain,

The domain is R\{1, 1}. The function is continuous everywhere except for the points
not part of the domain.
Exercise 3.1.3. For any real x, x2 + 1 is defined and nonzero. And, 1 + x2 is continuous
on R, so by theorem 3.1.9(d), it is continuous.
Exercise 3.1.4. Show that the function f (x) = |x| is continuous on all of R.
Proof. We need to find a such that for any  > 0, we have ||x| |a|| <  whenever
|x a| < .
Exercise 3.1.5. Assuming sin is continous, prove that sin(x3 4x) is continuous.
Proof. We know that |sin(x)| < 1 for all x.
Exercise 3.1.6. Since f (x), g(x) are continuous, and {f (xn )} converges to f (a) and {g(xn )}
converges to g(a), when {xn } converges to a. By theorem 2.3.6(d), {f (xn )/g(xn )} converges
to f (a)/g(a) since g(a) 6= 0 by assumption and g(xn ) 6= 0 in the domain of f (x)/g(x) by
definition.
25

26

CHAPTER 3. CONTINUOUS FUNCTIONS

Exercise 3.1.7. Let {xn } by a sequence in Df g . Then xn Dg so if xn a, g(xn ) g(a).


But all the xn , a Df g implies xn , a Dg and g(xn ), g(a) Df . Hence {g(xn )} is a sequence
in Df which converges to g(a) when {xn } converges to a, hence {f (g(xn ))} converges to
{f (g(a))}, so by theorem 3.1.6 it is continuous.

Exercise 3.1.8. We know x is continuous at all a 0 by theorem 3.1.7. Give another


proof of this fact by using only the definition of continuity.
Proof. We need to
between two cases:

distinguish

Case
1
a
=
0:
|
x

0|
=
x <  iff 0 x < 2 , = 2 . Whenever x < 2 we find that

x <  and therefore x is continuous at a = 0.

|xa|

Case 2 - a > 0: |x a| = | x a|| x + a|. This implies | x a| = xa
<
x
a
a

if we have |x a| <  a, =  a.
Exercise 3.1.9. Consider the function:

f (x) =

1
: x 0,
1 : x < 0

Is this function continuous if its domain is R? Is it continous if its domain is cut down to
{x R : x 0}? How about if its domain is {x R : x 0}?
Exercise 3.1.10. Let f be a function with domain D and suppose f is continuous at some
point a D. Prove that, for each  > 0, there is a > 0 such that
|f (x) f (y)| <  whenever x, y D (a , a + )
Exercise 3.1.11. Prove that the function
(
sin(1/x) x 6= 0
f (x) =
0
x=0
is not continuous at 0.
Proof. Whenever xn 0 we have f (xn ) f (0) = 0. We are looking for a sequence
1
. This goes to 0 but
xn 0 but where f (xn ) 6 f (0) = 0. We choose xn = /2+2n

sin( 1 + 2n) = 1.
Exercise 3.1.12. Prove that the function
(
x sin(1/x) x 6= 0
f (x) =
0
x=0
is continuous at 0.
Proof. We need to estimate |f (x) f (0)|.
 
 
1
1
|f (x) f (0)| = |x sin
| = |x|| sin
| |x| < 
x
x
Thus |x 0| <  for = .

3.2. PROPERTIES OF CONTINUOUS FUNCTIONS

27

Exercise 3.1.8. We know x is continuous at all a 0, by Theorem 3.1.7. Give another


proof of this fact using only the definition of continuity (Def. 3.1.1).
solved in class
Exercise 3.1.11. Prove that the function (piecewise function) is not continuous at 0.
solved in class
Exercise 3.1.12. Prove that the function (piecewise function) is continuous at 0.
solved in class

3.2

Properties of Continuous Functions

Exercise 3.2.1. The minimum of f on [0, 3) is 1. The maximum does not exist, since
sup f[0,3) = 6 which is achieved at x = 3 6 [0, 3).
Exercise 3.2.2. Prove that if f is a continuous function on a closed bounded interval I and
if f (x) is never 0 for x I, then there is a number m > 0 such that f (x) m for all x I
or f (x) m for all x I.
Proof. Assume f (a) > 0. We have that f ([a, b]) = [m, M ]. We know that m = min f ,
M = max f . Lets prove that m > 0. By contradiction: assume m < 0. Value 0 is taken
(non-legible) an intermediate value [m, f (a)] which contradicts f (x) 6= 0. Prove for case 2 is
analogous (show that M < 0).
Exercise 3.2.3. Prove that if f is a continuous function on a closed bounded interval [a, b]
and if (x0 , y0 ) is any point in the plane, then there is a closest point to (x0 , y0 ) on the graph
of f .
Proof. Pick any point x [a, b]. Then the distance to x0 , y0 is dist ((x0 , yo ), (x, f (x))) =
1
((x x0 )2 + (y0 f (x))2 )) 2 . We must prove that this function attains its minimum value
1
in [a, b] and that if f is continuous, then ((x x0 )2 + (y0 f (x))2 )) 2 is also continuous on
[a, b]. Then distance takes its minimum value there.
Exercise 3.2.4. Find an example of a function which is continuous on a bounded (but not
closed) interval I, but is not bounded. Then find an example of a function which is continuous and bounded on a bounded interval I, but does not have a maximum value.
The function f : (0, 1) R with f (x) = x1 fulfills the first condition.
The second condition cannot be fulfilled; according to theorem 3.2.1 (p. 65): If f is a
continuous function on a closed bounded interval I, then f is bounded on I and in fact, it
assumes both a minimum and a maximum value on I. The only way to create a function

28

CHAPTER 3. CONTINUOUS FUNCTIONS

which would not assume a maximum on such an interval would be by violating the continuity.
For example, the function
(
2x x < 1/2
f (x) =
0
x 1/2
fails to achieve its maximum on a bounded interval [0, 1]. However, it does so by having a
discontinuity at x = 1/2.
Exercise 3.2.5. Let f (x) = ex , I = [1, ). Then f is continuous on a closed (but not
bounded) interval, but is not bounded. If f (x) = 1/(1 + ex ) and I = [0, ), then f is
continuous and bounded by 1 on I which is closed but does not have a maximum.
Exercise 3.2.7. Give an example of a function defined on the interval [0, 1] which does not
take on every value between f (0) and f (1).
In other words, we are looking for a function with a discontinuity between [0, 1]. One
example would be:

x : 0 x < 21
f (x) =
2x : 12 x 1
Exercise 3.2.8. Show that if f and g are continuous functions on the interval [a, b] such
that f (a) < g(a) and g(b) < f (b), then there is a number c (a, b) such that f (c) = g(c).
Proof. We create a function h(x) = f (x) g(x). This is continuous since it is a linear
combination of continuous functions, and it is defined on [a, b]. We know that h(a) =
f (a) g(a) < 0 and h(b) = f (b) g(b) > 0. Bt the intermediate value theorem there exists
a c such that h(c) = f (c) g(c) = 0, which implies f (c) = g(c).
Exercise 3.2.9. Let f be a continuous function from [0, 1] to [0, 1].Prove that there is a
point c [0, 1] such that f (c) = c - that is, show that f has a fixed point. Hint: Apply the
Intermediate Value Theorem to the function g(x) = f (x) x.
Proof. Let g(x) = f (x) x. Since f (x) is continuous, we know that g(x) is also continuous.
Then g(a) 0 and g(b) 0. By the intermediate value theorem we know that there exists
some x [0, 1] such that g(x) = 0, which implies that f (x) = x.
Exercise 3.2.10. Use the intermediate value theorem to prove that if n is a natural number,
then every positive number a has a positive n-th root.
Proof. We write the function f (x) = xn which is continuous on [0, ) since it is a polynomial.
We notice f (0) = 0 < a. We know that there is a number m N such that m > a which
implies f (m) = mn m > a. Thus we have f (0) < a and f (m) > a and since f is
continuous on [0, m], the intermediate value theorem states that there exists a c such that
f (c) = cn = a.
Exercise 3.2.11. Prove that a polynomial of odd degree has at least one real root.

3.3. UNIFORM CONTINUITY

29

P
Proof. Assume g(x) : R R is an odd degree polynomial. Then g is of the form nk=0 ak xk ,
where ak is the k-th coefficient of the polynomial
We can
Pn1an nxk N such that n mod 2 =P1.n1
k
n
then factor g to be of the form g(x) = x (an + k=0 ak xn ). We note that limx k=0 ak xxn =
0. We then consider limx xn an . We note that since n is odd, xn 0 if x 0 and xn 0
if x 0. Therefore, limx+ xn an = + and limx xn an = , provided that an > 0.
Hence, we find that limx g(x) = and limx g(x) = . In the case of an < 0, we
find that limx g(x) = and limx g(x) = .
We know that any polynomial is continous, and the above shows that there are some a, b R
such that g(a) < 0 and g(b) > 0. We now consider the interval [a, b]. By the Intermediate
value theorem, we find that for every c [g(a), g(b)] there exists some x [a, b] such that
g(x) = c, implying that there exists at least one x such that g(x) = 0.
Exercise 3.2.12. Use the Intermediate Value Theorem to prove that f is a continuous
function on an interval [a, b] and if f (x) m for every x [a, b), then f (b) m.
Proof. Assume that m < f (b), such that f (x) m < f (b) for all x [a, b). We then know
that m = f (b) for some > 0 R. But, by properties of the real numbers, we would
also have m = f (b) < f (b)  < f (b) some some , such as  = 2 . But f (b)  [a, b)
- contradiction: by the intermediate value theorem, since f is continuous, we know that
there exists some x such that f (x) = f (b) , and thus we require m f (b) . Hence,
f (b) m.

3.3

Uniform Continuity

Exercise 3.3.1. Is the function f (x) = x2 uniformly continuous on (0, 1)? Justify your
answer.
Yes, it is. According to Theorem 3.3.4, if a function is continuous on a closed bounded
interval I, it is uniformly continuous there. Assume I = [0, 1]. Then by theorem 3.3.4, f is
uniformly continuous on I. By theorem 3.3.6, f is then also uniformly continuous on (0, 1).
Exercise 3.3.2. Is the function f (x) = 1/x2 uniformly continuous on (0, +) (actual text
printed asks only about interval up to 1 )? Justify your answer.
Assume f were uniformly continuous. Then it it is uniformly continuous on a subinterval,
such as (0, 1). But f (x) is not bounded on the interval (0, 1). Therefore, it is not uniformly
continuous.
Exercise 3.3.3. Is the function f (x) = x2 uniformly continuous on (0, +)? Justify your
answer.
No, it its not. As x we find that the distance between y, y 0 gets bigger and bigger,
such that x, x0 need to be closer and closer for y, y 0 to still be within  of each other. This
means that does depend on a, so it is not uniformly continuous.

30

CHAPTER 3. CONTINUOUS FUNCTIONS

Exercise 3.3.4. Using only the  definition of uniform continuity, prove that the function
x
is uniformly continuous on [0, ).
f (x) = x+1
Proof.
x
y
x(y + 1) + y(x + 1)

|=|
|
x+1 y+1
(x + 1)(y + 1)
xy + x xy y
|x y|
=|
|=
|x y|
(x + 1)(y + 1)
(x + 1)(y + 1)

|f (x) f (y)| = |

Estimate |f (x) f (y)| |x y|. Then for all  > 0, =  implies |x y| < =  will result
in |f (x) f (y)| < .
Exercise 3.3.5. In example 3.3.8 we showed that
Show that it is also uniformly continuous on [0, 1].
By theorem 3.3.4: if

x is uniformly continuous on [1, ).

x is continuous on [0, 1], it is uniformly continuous there.

Exercise 3.3.6. Prove that if I and J are overlapping intervals in R(I J 6= and f is a
function, defined on I J, which is uniformly continuous on I and uniformly continuous on
J, then
it is also uniformly continuous on I J. Use this and the previous exercise to prove
that x is uniformly continuous on [0, +).
Proof. By assumption I J 6= . We shall assume that the interval I is the lower one of
the two. Then there exists an x such that x I J. Since I is uniformly continuous by
assumption, we know that [x a, x] is uniformly continous for all (x a) I. Likewise we
know that [x, x + b] is uniformly continous for all (x + b) J since J is uniformly continous
by assumption.This implies that the whole interval [x a, x + b] is uniformly continous.
we are given some  > 0. For this, we
To prove that x is uniformly
continuous,
we assume

pick = 2 . We note that | x y| | x + y|. If |x y| < = 2 , we find:

| x y|2 | x y|| x + y| = |x y| < 2

This guarantees that | x y| < , proving that x is uniformly continous on (0, ).


Exercise 3.3.7. Suppose f is not continuous on I, then since uniform continuity implies
continuity, f is not uniform continuous on I. Suppose f is continuous on I. Then f is
But a continuous function on a closed,
uniform continuous on I if f is continuouss on I.
bounded interval is bounded, but f is unbounded. This is a contradiction so f cannot be
uniformly continuous.
Exercise 3.3.8. Let f be a function defined on an interval I and suppose that there are
positive constants K and r such that
|f (x) f (y)| K|x y|r for all x, y I.
Prove that f is uniformly continuous.

3.4. UNIFORM CONVERGENCE

31

Proof. According to assumption, we find that |f (x) f (y)| K|x y|r for all x, y I.
This implies that if K|x y|r < , |f (x) f (y)| < . Thus we find that we p
need to solve
r
K|x y| < , and find that for any given , we pick a such that = r K . Since
does not depend on where x, y are in the interval, f is uniformly continuous.
Exercise 3.3.9. Is the function f (x) = sin( x1 ) continuous on (0, 1)? Is it uniformly continuous on (0, 1)? Justify your answers.
Proof. Since sin is a trigonometric function, it is continuous on its whole domain. Likewise,
sin(1/x) is continous since it is merely a composition of two elementary functions.
However, sin(1/x) is not uniformly continous. The reason for this is that the as x 0
the function oscillates between 1 and 1. Thus, a that would work at one point in the
function will can produce potentially a difference |f (x) f (y)| = 2 for x, y sufficiently close
to 0. Hence, the functions is not uniformly continous.
Exercise 3.3.10. Is the function f (x) = x sin(1/x) uniformly continuous on (0, 1)? Justify
your answer.
Proof. Method 1: f (1) = sin(1). It is still uniformly continuous. limx0 f (x) = limx0 x sin(1/x) =
0. By squeeze theorem:
0 by squeeze thrm.

}|
 {
1
|
|x sin
x
z

0
|{z}
0

|x|
|{z}
0

If we define f (0) = 0, f (1) = sin(1), then f (x) becomes continuous on [0, 1]. then by
theorem 3.3.4, f is uniformly continuous on [0, 1] = f is uniformly continuous on (0, 1).
Method 2:
|f (x) f (y)| = |x sin(1/x) y sin(1/y)| |x sin(1/x)| + |y sin(1/y)| |x| + |y|
Then for all  > 0 we have |f (x) f (y)| <  if x, y (0, 2 ]. If now x, y > /3, then there
exists a > 0 such that |f (x) f (y)| <  whenever |x y| < :
|f (x) f (y)| |x| + |y| <




+ |x| + |y x| < + + < 
3
3 3

if < 3 . Then we choose = min( 3 , 0 ).

3.4

Uniform Convergence

32

CHAPTER 3. CONTINUOUS FUNCTIONS

Chapter 4
The Derivative
4.1

Limits of Functions

4.2

The Derivative

4.3

The Mean Value Theorem

Exercise 4.3.1.
Exercise 4.3.1. If we apply the MVT to the intervals [1, 1], [0, 1] and [1, 0] we find
that there exist points where the derivative equals 1/2, 1, and 0.
Exercise 4.3.2. Observe that f (x) = sin x satisfies the hypotheses of theorem 4.3.9
where we let (a, b) = (, ) and so it must satisfy |f (x) f (y)| M |x y| where M is a
bound for the derivative. In this case f 0 (x) = cos x 1 for all x, hence we may set M = 1.
Then | sin x sin y| = |f (x) f (y)| M |x y| = |x y| as desired.
Exercise 4.3.3.
Exercise 4.3.4. We know that there is some c (0, ) with f (x)f (y) = f 0 (c)(xy) for
x, y (0, ). Then |f (x) f (y)| f 0 (c)|x y| M |x y|. If we take the limit of both sides
as y 0, then since |f (x) f (y)| is continuous, lim |f (x) f (y)| = |f (x) f (0)| = |f (x)|.
y0

Hence |f (x)| M x.
Exercise 4.3.5.
Exercise 4.3.6. We know that f 0 (x) = 6x2 + 6x 12 = 6(x + 2)(x 1). Hence f 0 (x) 0
on [2, 1] and is positive elsewhere. Hence f (x) is decreasing on [2, 1] and increasing
elsewhere.
Exercise 4.3.7.
Exercise 4.3.8.
33

34

CHAPTER 4. THE DERIVATIVE

Exercise 4.3.9.
Exercise 4.3.10.
Exercise 4.3.11.
Exercise 4.3.12.
Exercise 4.3.13.
Exercise 4.3.14.
Exercise 4.3.15.
Exercise 4.3.16.

4.4

LHopitals Rule

Exercise 4.4.1. By Cauchys form of the MVT for the interval [1, x] and f (x) = ln x and
g(x) = xr , we have
ln x
1/c
1
= r1 = r ,
r
x 1
rc
rc
hence
xr 1
ln x =
.
rcr
We know moreover that c > 1 and r > 0, hence r logc 1, or cr 1 hence 1/cr 1. Then
ln x =

xr 1
xr 1

,
rcr
r

as desired.
Exercise 4.4.2.
Exercise 4.4.3.
Exercise 4.4.4.
Exercise 4.4.5.
Exercise 4.4.6. The limit is of the form /, hence
ln x
1/x
=
lim
x rxr1
x xr
1
= lim
= 0.
x rxr
lim

4.4. LHOPITALS RULE

35

Exercise 4.4.7. If we write x ln x =

ln x
then the limit is of the form /. By
1/x

LHopitals, the limit is equal to


1/x
= lim x = 0.
x0 1/x2
x0
lim

Exercise 4.4.8. The limit is of the form 0/0 hence it is equal to


cos x 1
.
x0
3x2
lim

It is still of the form 0/0 hence equal to


lim

x0

sin x
.
6x

Applying LHopitals again, the limit equals


lim

x0

cos x
= 0.
6

Exercise 4.4.9.
Exercise 4.4.10. Let g(x) = log f (x) = x log x. Then
lim x log x = lim log x/(1/x).

x0

x0

This has numerator and denominator , hence we may apply LHopitals to get
lim x log x = lim (1/x)/(1/x2 ) = lim x = 0.

x0

Exercise 4.4.11.
Exercise 4.4.12.
Exercise 4.4.13.
Exercise 4.4.14.
Exercise 4.4.15.
Exercise 4.4.16.

x0

x0

36

CHAPTER 4. THE DERIVATIVE

Chapter 5
The Integral
5.1

Definition of the Integral

Exercise 5.1.1. We have


U (f, P ) = 1

1 4 1 2 1 4 1
+ + + = 319/40.
4 5 4 3 4 7 4

Moreover,
L(f, P ) =

4 1 2 1 4 1 1 1
+ + + = 533/840.
5 4 3 4 7 4 2 4

Exercise 5.1.2. Since x is an increasing function, we know that Mk = k/n and mk =


(k 1)/n hence
n
X
k1
1 n(n + 1)
n+1
U (f, Pn ) =
= 2
=
nn
n
2
2n
k=1
and
L(f, Pn ) =

n
X
k11
k=1

1 n(n 1)
n1
=
.
2
n
2
2n

Since these have the same limit as n , we know that limn U (f, Pn ) L(f, Pn ) = 0,
as desired.
Exercise 5.1.3. The base case is clearly true, so suppose that
k
X

j2 =

j=1

k(k + 1)(2k + 1)
.
6

Then adding (k + 1)2 to both sides yields


k+1
X
j=1

j2 =

k(k + 1)(2k + 1)
(k + 1)(k + 2)(2(k + 1) + 1)
+ (k + 1)2 =
,
6
6

as desired.
37

38

CHAPTER 5. THE INTEGRAL


Exercise 5.1.4. By using exercise 3 above, one finds that
2
n 
X
ka
a
a3 n(n + 1)(2n + 1)
U (f, Pn ) =
= 2
,
n
n
n
6
k=1

(n 1)a
a
< <
< a} and Mk = (ka/n)2 since f (x) = x2 is increasing.
n
n
Moreover, notice that the limit is a3 /3. We now find that
2
n 
X
(k 1)a
a
L(f, Pn ) =
n
n
k=1

where Pn = {0 <

which we may find evaluates to


a3 n(n 1)(2n 1)
.
n2
6
Notice that this also has limit a3 /3, hence the limit of the difference of the two is zero, and
the integral exists. Moreover, it equals lim U (f, Pn ) = a3 /3.
X
Mk (xk
Exercise 5.1.5. For any P , we know that U (f, P ) = 1 since U (f, P ) =
k
X
xk1 ) =
xk xk1 = 1 0 = 1 as the supremum of f on any interval is 1. On the
k

other hand we know L(f, P ) = 0 since the infimum of f on any interval is 0. Hence for all
partitions P , U (f, P ) L(f, P ) = 1, hence by theorem 5.1.7 the function is not Riemann
integrable.
Exercise 5.1.8. We know that
Z b
f (x) dx = sup{L(f, Q) : Q P},
a

P
where P is the set of partitions of [a, b]. This is equal to nk=1 mk (xk xk1 ) for some
partition {a = x0 < x1 < < xn = b} of [a, b]. But we know
n
X

mk (xk xk1 )

k=1

n
X

m(xk xk1 ) = m(b a).

k=1

This gives the first inequality in the chain of inequalities. The second inequality is obtained
by the definition of supremum and infimum. The third inequality is found by noting that
Mk M .
Exercise 5.1.9. For any partition,
U (f, P ) =

n
X
k=1

Mj (xj xj1 ) =

n
X
j=1

k(b a) =

n
X

mk (b a) = L(f, P ).

j=1

Hence U (f, P ) L(f, P ) = 0 <  for any partition P , so by theorem 5.1.7 the integral exists.
Applying definition
Pn 5.1.6 and the definition of upper and lower integral, we find that the
integral equals j=1 k(b a).

5.2. EXISTENCE AND PROPERTIES OF THE INTEGRAL

39

Exercise 5.1.10. See definition 5.1.1. If P partitions [a, b] into n equal subintervals,
then xk xk1 = (a b)/n hence the difference is
n

abX
(Mk mk ).
n k=1

5.2

Existence and Properties of the Integral

Exercise 5.2.1. If f = g h and g, h are non-decreasing, then g 0 , h0 0. We will then have


g 0 h0 , g 0 = h0 or g 0 h0 . In the first and third cases, we know that either f 0 0 or f 0 0.
In either case, f is monotone on a closed bounded interval [a, b], hence by theorem 5.2.1 it is
integrable. On the other hand if g 0 = h0 then f 0 = 0, hence f is constant and it was proved
that a constant function is integrable in the previous homework.
Exercise 5.2.6. We know 1 + x2n is continuous and never zero, hence 1/(1 + x2n ) is also
continuous on [1, 1], hence by theorem 5.2.2 it is integrable. Moreover, M = sup[1,1] f = 1
and m = inf [1,1] f = 1/2. By corollary 5.2.5, we know that the integral is therefore trapped
between m(b a) = 12 2 = 1 and M (b a) = 1 2 = 2. Hence the desired inequality.
Exercise 5.2.11. Let f (x) = 1 if x [0, 1] is rational, and 1 if x is irrational. Then
for reasons similar to problem 5.1.5, this function is not integrable. However, |f (x)| = 1 for
all x [0, 1], and hence it is integrable.
Exercise 5.2.12. We know that f (x) is continuous, hence integrable on a closed bounded
interval [a, b]. It is also therefore bounded, and so M = sup[a,b] f and m = inf [a,b] f are finite
so f actually attains these values. Moreover, we know by the IVT that f attains every value
between m and M (possibly these values). In particular, f attains the value
Z b
1
f (x) dx,
ba a
by corollary 5.2.5.

5.3

The Fundamental Theorems of Calculus

Exercise 5.3.1. We know that the antiderivative is f (x) = x2 sin(1/x) and that it satisfies
all the hypotheses
of theorem 5.3.1 from example 5.3.2, and so the integral equals f (2/)
2
f (4/) = 4/ 8 2/ 2 .
Exercise 5.3.2. It is clear that the derivative is cos(1/x).
Exercise 5.3.4. The integral may be split up as
Z x
Z 1/x
2
t2
e
dt
et dt,
0

0
2

and hence the derivative of the first integral is ex and of the second it is e1/x /x2 by
the chain rule and second fundamental theorem of calculus.
Exercise 5.3.5. f (x) = 1/x is not continuous at x = 0 [1, 1], violating the
hypotheses of theorem 5.3.1.

40

CHAPTER 5. THE INTEGRAL

Exercise 5.3.6. Letting g(x) = f (x), applying (5.3.5) gives that the integral equals
Z b
2
2
(f 0 (x))2 dx.
f (b) f (a)
a

1 n+1
t
and
n+1
f 0 (t) = 1/t. Both f and g are continuous on [0, ) and differentiable on (0, ). We know
f g 0 and gf 0 are continuous, hence by (5.3.5) the integral equals
x Z x

1 n+1 1
1 n+1
t
ln t
t
dt.
n+1
t
0 n+1
0
Exercise 5.3.8. Let x t and g 0 (t) = tn and f (t) = ln t. Then g(t) =

Simplifying this and making the necessary evaluations, the integral comes out as
1
1
xn+1 ln x
xn+1 .
n+1
(n + 1)2
Exercise 5.3.10. Theorem 5.3.1 requires f (x) = x/|x| be continuous at every point in
[1, 1], but f (x) is not continuous at x = 0.

5.4

Logs, Exponentials, Improper Integrals

Chapter 6
Infinite Series
6.1

Convergence of Infinite Series

6.2

Tests for Convergence

6.3

Absolute and Conditional Convergence

6.4

Power Series

Exercise 6.4.2. Prove that f (x) =

k=1

sin(kx)
2k

is continuous on the entire real line.

Proof. We use the Ratio test to show that the sum converges on the entire real line:
sin ((k + 1)x) 2k
sin (kx + x)
1
=

.
2k 2
sin(kx)
2 sin(kx)
2
P
k
= 2,
This proves convergence on all of R. We notice that sin(kx)2k 2k . Since
k=0 2
P sin(kx)
we know by the Weierstrass M-test that k=1 2k converges uniformly on all R. This
proves that this sum is continuous on the entire real line.
Exercise 6.4.4. Radius of convergence of

1
k
k=1 k3k x .

We have ck = (k3k )1 . We proceed to take the k-th root:


(k3k )1/k =

1 1
= 31 .
3 kk

We then solve using the formula for the radius of convergence:


R=

1
1
=
= 3.

lim sup k ck
lim sup 31

Therefore, the radius of convergence is 3.


41

42

CHAPTER 6. INFINITE SERIES

Exercise 6.4.5. Radius of convergence of


We have ck =

(1)k1
.
k+1

k=0

(1)k1
(x
k+1

+ 2)k .

We take the k-th root:

1/k

(ck )

1 
1
(1)k1 k
(1)k k
=
=
k+1
k+1
1
1
(1)k/k
=
=
.
=
1/k
1/k
(k + 1)
(k + 1)
(k + 1)1/k


We then use the formula for the radius of convergence:


R=

1
1
1
=
= = 1.

1/k
k
lim sup ck
1
lim sup (k + 1)

And hence, the radius of convergence is 1.


Exercise 6.4.8. Radius of convergence of

k=0

2k x2k .

We have ck = 2k . We then take the k-th root: 2k


the the radius of convergence:
R=

 k1

= 2k/k = 2. We use the formula for

1
1
1
= .
=

k
lim sup ck
lim sup 2
2

Thus we can see that the radius of convergence is 1/2.


Exercise 6.4.10. Let {ak } be a non-increasing sequence of non-negative
numbers which
P
k+1
converges to 0. Use theorem 6.3.2 to show that the power series k=0 (1) ak xk converges
uniformly on [0, 1], and hence converges to a continuous function on this interval.
P
k+1
ak converges provided that {ak }
Proof. According to theorem 6.3.2, the series
k=0 (1)
is a non-increasing sequence of non-negative numbers. We notice that this power series is
uniformly Cauchy, since the sequence of partial sums is uniformly Cauchy according to the
theorem. Therefore, this series converges uniformly.
k+1
Notice
x [0, 1], (1)P
ak xk (1)k+1 ak . Therefore, we see that the series
P thatk+1for all

(1) ak xk is bounded by k=0 (1)k+1 ak , which by the Weierstrass M-test implies


k=0P
k+1
that
ak xk is uniformly convergent on [0, 1].
k=0 (1)
Exercise 6.4.12. Prove that if f (x) is the sum of a power series centered at a and with
radius of convergence R, then f is infinitely differentiable on (a R, a + R) - that is, its
derivative of order m exists on this interval for all m N.
P
k
Proof. Suppose that f (x) =
to theorem 6.4.12, f (x)
k=0 ck (x a) . Then, according
P
k1
is differentiable on (a R, a + R) and its derivative is f 0 (x) =
. This
k=1 kck (x a)
means we are left with a k 1 degree differentiable polynomial. By induction we see that
we can then integrate this power series again. Since we take k to infinity, this means we can
take an infinite number of derivatives. Additionally, any polynomial has an infinite number
of derivatives: the kth derivative is a constant, and all others are zero.

6.5. TAYLORS FORMULA

6.5

43

Taylors Formula
n

Exercise 6.5.1. Prove that lim xn! = 0 for all x.


Proof. Since we take the limit as n , n will take on every value of N. We assume that
x R. Therefore, at some point we find have that x n; at the first point this occurs, we
xn1
. If we
then consider the rational expression nx C where C is some constant of the form (n1)!
xx
consier the next term in the sequence, n (n+1) C, we find that n (n + 1) x x, since x n.
n
Therefore, it is obvious that limn xn! = 0 for all x.
Exercise 6.5.3. Use Taylors formula to estimate the error if cos(x) is approximated by
2
1 x2 on the interval [0.1, 0.1].
We estimate as follows:
0.14
x4

5 106
4!
4!
Thus we can bound the error by 5 106 .
Exercise 6.5.5. Taylors formula for f (x) =

t + x with a = 0.

We begin by taking the first few derivatives of f (x) =

t + x.

1
f 0 (x) =
2 t+x
1
f 00 (x) =
3
4 (t + x) 2
3
f (3) (x) =
5
8 (t + x) 2
15
f (4) (x) =
7
16 (t + x) 2
Then the Taylor formula for the above function for the first 5 terms, with remainder, at
a = 0 will be as follows:
f (x) =

X
f (n) (a)
n=0

n!

(x a)n

1
1 1
3 1
15 1 4
105
1 5
t + x 3 x 2 + 5 x3
x +
x
9
7
2 t
4t 2 2
8t 2 3!
16t 2 4!
32 (c + t) 2 5!

x
1
1 3
5
7
4
5
= t + 3 x2 +
5 x
7 x +
9 x .
2 t 8t 2
16t 2
128t 2
256 (c + t) 2
Exercise 6.5.7. Taylors formula for f (x) = ln(1 + x) with a = 0.

44

CHAPTER 6. INFINITE SERIES


We again begin by taking the first four derivatives of f .
f (x) = ln(1 + x)
1
f 0 (x) =
x+1
1
f 00 (x) =
(x + 1)2
2
f (3) (x) =
(x + 1)3
6
f (4) (x) =
(x + 1)4

Then the Taylor formula for the above function for the first 5 terms, with remainder, at
a = 0 will be as follows:
f (x) =

X
f (n) (a)

n!

n=0

1
x x2 +
2
1
= x x2 +
2

(x a)n
2 3
x
6
1 3
x
3

6 4
24
5
x +
5x
24
120 (c + 1)
1 4
1
5
x +
5x .
4
5 (c + 1)

Exercise 6.5.11. If f is an infinitely


function on (a r, a + r) and there is
differentiable
n!
(n)


some constant K such that f (x) K rn for all n N and all x (a r, a + r), then the
Taylor series for f at a converges to f on (a r, a + r).
This appears to be a backwards reasoning argument. We that if f is a convergent taylor
series on (a R, a + R), then it is infitinely differentiable on that same interval.
We can see that if there is some constant K such that f (n) (x) K rn!n for all n N and
all x (a r, a + r), then f (n) (x) is always bounded.
2

Exercise 6.5.13. If g(x) = e1/x for x 6= 0 and g(0) = 0, show that g is infinitely differentiable on the entire real line but all of its derivatives at 0 are 0. Argue that this means that
g cannot be analytic at 0. Hint: Use the previous exercise to help compute the derivatives of
g at 0.
1/x2

First note that g 0 (x) = 2 e x3 . The Taylor expansion of e1/x centered around a is
P
(k)
f (k) (a)
of the form
(x a)k . This is a power series where ck = f k!(a) and as such is
k=0
k!
differentiable according to theorem 6.4.12, and infinitely so (cf. exercise 6.4.12). Likewise,
2
e1/x is infinitely differentiable, and all of its diefferentials are clearly defined everywhere
except
at x = 0. This is due to the fact that all n-th derivatives of g are sums of the form
Pn
1/x2 (n+k+1)
a
x
. Therefore, analytically g(x) is not defined at 0.
k=1 k e
1/x2

Remember however that limx0 e xn = 0. Therefore, if we define g(x) and and all of its
derivatives to be 0 at 0, g(x) is infinitely differentiable.

Part II
Multivariable - 3220

45

Chapter 7
Convergence in Euclidean Space
7.1

Euclidean Space

Exercise 7.1.5. Prove that ||x| |y|| |x y|.


Proof. We note that x = x y + y. This implies that |x| = |x y + y|. We use the triangle
inequality:
|x y + y| |x y| + |y|
|x| |x y| + |y|
|x| |y| |x y|

which is equivalent to
| |y|

This proves the result.


Exercise 7.1.6. Prove that equality holds in the Cauchy-Schwartz inequality if and only if
one of the vectors ~u, ~v is a scalar multiple of the other.
Proof. Let ~v be a scalar multiple of ~u, such that ~v = ~u. Then
~u ~v = ~u ~u = u21 + u22 + . . . + u2n

= u21 + . . . + u2n
2
v
u n
uX
u2i = |~u|2 .
= t
i=1

Therefore, we find that |~u|2 |~v |2 = |~u| |~u|2 = |~u|2 , and hence, equality holds. The proof
for the case where ~u is a scalar multiple of ~v is analogous.

7.2

Convergent Sequences of Vectors

7.3

Open and Closed Sets

Exercise 7.3.1. Prove that the set {(x, y) R2 : y > 0} is an open subset of R2 .
47

48

CHAPTER 7. CONVERGENCE IN EUCLIDEAN SPACE

Proof. According to theorem 7.3.10, a set A Rd is closed if and only if every convergent
sequence in A converges to a point x A. Consider the sequence { n1 , n1 }. This sequence
is clearly contained in the set A = {(x, y) R2 : y > 0}. However, { n1 , n1 } (0, 0), and
(0, 0)
/ A. Therefore, the set A is not closed. Since A is not closed, A is an open set.
Exercise 7.3.4. Find the interior, closure, and boundary for the set
A = {(x, y) R2 : |(x, y)| < 1} {(x, y) R2 : y = 0, 2 < x < 2}.
The solution is as follows:
A = {(x, y) R2 : |(x, y)| < 1} {(x, y) R2 : y = 0, 2 < x < 2}
A = {(x, y) R2 : |(x, y)| 1 y = 0, |x| 2}
A = {(x, y) R2 : |(x, y)| = 1 x = 2, y = 0 x = 2, y = 0}
Exercise 7.3.6. Let A be an open set and B a closed set. If B A, prove that A\B is
open. If A B, prove that B\A is closed.
Proof. Case 1: B A. Since B is a closed set, for any sequence {xn } B, there is some
Li B such that {xn } Li . Since B A, we know that {xn }, Li A for all n, i N.
If B is a proper subset of A, then for every Li there exists some sequence {yn } such that
{yn } A\B and {yn } Li . Since Li B, we find that Li
/ A\B. But then not all limit
points of A\B are actually in A\B, thus showing that A\B is an open set.
Case 2: A B. Since A is an open set, Li , {xn } such that Li B, {xn } A and
{xn } Li . Consider now the set B\A. Since Li B, but Li
/ A, we find that Li B\A i.
Therefore, B\A is a closed set.
Exercise 7.3.9. If A and B are subsets of Rd show that A B = A B. Is the analogous
statement true for A B? Justify your answer.
Existing solution utilizes a graphic. Try to find analytic solution instead.
Exercise 7.3.15. If E is a subset of Rd , show that (E)c = (E c ) .
As is evident from Figure 1, this identity holds.

7.4

Compact Sets

d
Exercise 7.4.1. If K is a compact subset
Sof R and U1 U2 . . . Uk . . . is a nested
upward sequence of open sets with K k Uk , then prove that K is contained in one of
the sets Uk .

Proof. This seems obvious.


Since UK is a nested upward sequence,
we have that Ui Ui+1
S
S
for all i N . If K k Uk , then xi K we find that xi k Uk . Since K isSa compact
set, it has a finite subcoverage. This means, there is aSfinite n N such that n Un K.
Therefore, there exists a finite m N such that K n+m Um+n .

7.4. COMPACT SETS

49

Exercise 7.4.4. Prove that if K is a compact subset of Rd , then K contains a point of


maximal norm. That is, there is a point x1 K such that
|x| |x1 | for all x K.
Hint: set m = sup{|x| : x K} and consider the open balls Bm1/n (0).
Proof. A continuous image of a compact space is compact. Consider the space K Rd ,
and let f : K R such that f (x) = |x|. This is a continuous function since the space is
compact. As a continuous, real valued function, it attains both a maximum and a minimum
per 3.2.1. Therefore, there exists an xi K such that f (xi ) f (xk ) for all k N. This
proves that x : |x| |xi | for all i N.
Exercise 7.4.6. Prove that the conclusion of the previous exercise also holds if we only
assume that K is a closed subset of Rd . Hint: replace K by its intersection with a suitably
large closed ball centered at y.
Proof. Let f (x) : K R such that f (x) = |x y| where y Rd . Since K is a compact
set imaged into a compact set, we know that f is a continuous function. As such, this
function attains both a minimum and a maximum per 3.2.1. Therefore, there exists a point
xi such that f (xi ) f (xk ) for all k N, which proves that there exists a point xi such that
|xi y| |x y| for all x.
Exercise 7.4.9. Show that it is true that the union of any finite collection of compact subsets of Rd is compact, but it is not true that the union of an infinite collection of compact
subsets is necessarily compact. Show the latter statement by finding an example of an infinite union of compact sets which is not compact.
Any compact set has a finite subcoverage. Having a finite subcoverage - given an open
cover - is sufficient for a set to be considered compact. The union of 2 compact sets then has
two sets of compact subcoverages. The union of subcoverages is likewise finite. Assume we
have n compact sets with finite subcoverage. The union of these n sets with an additional
compact set results in the union of a set of finite subcoverages of the union of the n sets
together with a finite subcoverage of the n + 1-th set. This is then evidently also finite.
Hence, any finite union of compact sets is also compact. However, an infinite collection is
not necessarily compact as this infinite union may lead to the existence of an infinite number
of subcoverages, which would make this union no longer compact.
Exercise 7.4.10. Prove that if A and B are compact subsets of a Rd , then A B and A B
are also compact.
Proof. Any compact set has a finite open coverage. Hence, there is a finite open cover UA of
A and an open coverage UB of B. We now consider the union UA UB . Since both coverages
are finite, the union is finite as well. This union now is a finite coverage of A B. Since
A B has a finite coverage, it is also compact.

50

7.5

CHAPTER 7. CONVERGENCE IN EUCLIDEAN SPACE

Connected Sets

Exercise 7.5.1. A = {(x, y) R2 : |(x, y)| < 1} {(x, y) R2 : 1 x 2, y = 0}. This


set is connected, since it is the union of two connected sets whose intersect is non-empty.
Exercise 7.5.3. A = {(x, y) R2 : 1 < |(x, y)| < 2}. This set is an open set in the form of
a disk, and as such is connected.
Exercise 7.5.5. What are the connected components of the complement of the set of integers in R?
The connected components of this set consists of all sets A = {x R : m < x < m + 1}
for all m Z.
Exercise 7.5.7. Which subsets of R are both compact and connected? Justify.
Any closed interval in R will be both compact and connected. That is any set A = {x
R : m x n} for some m, n R. It is important that this interval not contain any
holes, as it is possible to create a compact set with holes which would not be connected. For
example, the Cantor set is considered to be both compact and totally disconnected.
Exercise 7.5.10. Prove that the closure of a connected set is connected.
Proof. We need to consider two distinct cases: either A is an open set, or a closed set. If A
is a closed set, then A = A and hence, the closure is connected.
If however A is an open set, then there exists at least one sequence {xn } such that xn A
for all n N, {xn } L, but L
/ A. Consider now the set A. Now, L A and A A.
Assume now that A is not a connected set. Then there exists some xi such that xi {xn }
but xi
/ A. But by assumption, xi {xn } and therefore xi A. Since A A, which shows
that xi A. Contradiction. Hence, A must also be a connected set.

Chapter 8
Functions on Euclidean Space
8.1

Continuous Functions of Several Variables

Exercise 8.1.2. Give a simple reason why the function : R R4 defined by (t) =
(t, sin(t), et , t2 ) is continuous in R.
The function consists solely of continuous functions in R, and as such (t) is likewise
continuous (theorem 8.1.5).
Exercise 8.1.4. Consider the function f : R2 R defined by

x y : x y > 0.
f (x, y) =
0
: x y 0.
at which points of R2 is this function continuous?
This function is continuous whenever
x, y > 0
x, y < 0
x < 0, y > 0
or x > 0, y < 0.
The function is also continuous at zero if we approach (0, 0) along the axis from the same
quadrant.

Exercise 8.1.5. For the function f : R2 R defined by


f (x, y) =
51

x2 y
x4 + y 2

52

CHAPTER 8. FUNCTIONS ON EUCLIDEAN SPACE

show that f has a limit 0 as (x, y) (0, 0) along any straight line through the origin but
that it does not have a limit as (x, y) (0, 0) in R2 .
Assume thatwe approach along the x = 0 axis. In that case, the function has the limit 0.
Assume now we approach along some line, which has the form y = mx. Then we consider
x2 (mx)
x3 m
mx
= lim
= lim
= 0.
lim
4
2
2
4
2
4
2
x,y(0,0) x + (mx )
x,y(0,0) x + m x
x,y(0,0) x + m2
The function does not however have a limit itself. For a function to have a limit, its limit
must be path-independent. This is not the case for the function in question. Assume you
approach via the path y = mx2 . The resulting limit
m
x4 m
x2 (mx2 )
= lim
=
lim
4
2
2
4
4
x,y(0,0) 1 + m
x,y(0,0) x + x m
x,y(0,0) x + (mx )
lim

clearly depends on m, and as such is not path-independent.


Exercise 8.1.12. Let B1 (0) be the open unit ball in R2 . Is it true that every continuous
function f : B1 (0) R takes Cauchy sequences to Cauchy sequences?
I cant come up with a counter example; as such, I think this holds true given that we
are considering an open ball which includes the point (0, 0). If we however choose the go
towards the functions via a path of the form y = mx2 .
Exercise 8.1.14. Find a parameterized curve (t) in R2 , with parameter interval [0, ),
that begins at (1, 0), spirals inward in the counterclockwise direction, and approaches (0, 0)
as t .
We use the function f (r, ) = (r cos(), r sin()).

8.2

Properties of Continuous Functions

Exercise 8.2.3. If K is a compact, connected subset of Rp and f : K R is continuous


function, what can you say about f (K)?
We know that a continuous function maps a compact set to a compact set, and that a
continuous function also maps a connected set to a connected set. As such, I would argue
that f (K) is a compact, connected set.
Exercise 8.2.5. The image of a compact set under a continuous function is compact, hence
closed by theorem 8.2.3. Is the image of a closed set under a continuous function necessarily
closed? Prove that it is or give a counter example.
A simple counter example would be the function f : R R+ where f (x) = ex . Then the
preimage of f is closed, yet the image of f is not, as the limit point limx ex = 0 is not
part of the image set.

8.3. SEQUENCES OF FUNCTIONS

53

Exercise 8.2.7. Is the sphere {(x, y, z) R3 : x2 + y 2 + z 2 = 1} connected? How do you


know?
The sphere described above is connected. We know that any path-connected set is a
connected set. The surface of a sphere is path connected, as we can join any two points
through a continuous path function. As such, I would argue that the sphere is simply
connected, which implies that it is a connected set.
Exercise 8.2.10. Is the function f : R2 \{(2, 0} R defined by
f (x, y) =

1
(x 2)2 + y 2

uniformly continuous on B1 (0, 0)? Is it uniformly continuous on B2 (0, 0)? Justify your answers.
I think it is uniformly continuous on B1 (0, 0), as well as on B2 (0, 0). In both cases, we
can bound the function appropriately such that it goes to 0 without depending on x and y.
Exercise 8.2.11. If D Rp , prove that if a function F : D Rq is uniformly continuous
on D, then {F (xn )} is a Cauchy sequence in Rq whenever {xn } is a Cauchy sequence in D.
Proof. Assume F is defined as above, with {xn } being a Cauchy sqeuence in D. Since F is
a a uniformly continuous mapping, we know that for all  > 0, there exists some > 0 such
that |f (xn ) f (xm )| <  whenever |xn xm | < . Since {xn } is Cauchy, we know that there
exists some N > 0 such that |xn xm | < whenever n, m > N . Due to uniform continuity,
we know that is independent of x. Hence, if |xn xm | < , we have |f (xn ) f (xm )| < 
for m, n > N . As such, the image of the Cauchy sequence is again Cauchy.

8.3

Sequences of Functions

Exercise 8.3.1. Show that the sequence {n (t)}, where




1
t
,
,
n (t) =
1 + nt n
does not converge uniformly on [0, 1].
For the function to converge uniformly, both parts of the function need to individually
converge uniformly. While it is true that t/n converges uniformly on [0, 1] since it can be
bound by 1/n, it is not true that 1/(1 + nt) converges uniformly. Specifically, we find that



1 1



1 + n 1 + nt 1.
As can be seen, this bound is dependent on t, since we need n large whenever t approaches
0 to be within  > 0. Hence, this part of the function fails to converge uniformly, which
results in our sequence n (t) failing to converge uniformly.

54

CHAPTER 8. FUNCTIONS ON EUCLIDEAN SPACE

Exercise 8.3.3. Does the sequence {(k 1 sin(kx), k 1 cos(ky))} converge pointwise in R2 ?
Does it converge uniformly in R2 ? Justify your answers.
o
n
1
1
. Clearly, the limit of this sequence
,
We can rewrite this sequence as
k csc(kx) k sec(ky)
is (0, 0) for all x. As both the secant and cosecant functions arent properly bounded, I
would argue that this convergence cannot be uniform, and must be pointwise.
P
k k
Exercise 8.3.8. Does the series
k=0 x y converge uniformly on the square


(x, y) R2 : 1 < x < 1, 1 < y < 1 ?
Justify your answer.
This series converges uniformly. It is an adaptation of the geometric series in multiple
variables. The geometric series exhibits uniform convergence per the Cauchy criterion. As
such, I would argue that the extension of the geometric series to R2 would converge uniformly
as well.
P
n
n
Exercise 8.3.10. Does the series
k=0 (x , (1 x) ) converge pointwise on [0, 1]? Does it
converge pointwise on (0, 1)? On which subsets of (0, 1) does it converge uniformly? Justify
your answers.
P
n
The series does not converge pointwise onP
[0, 1], since
k=0 1 does not converge. It does

n
however converge pointwise on (0, 1), since k=0 k converges pointwise provided |k| < 1.
I would argue that the series in question converges uniformly for all rightclosed
subsets of
P
n
x
passes
the
(0, 1), i.e. all sets of the form (0, a] where a <P1. It is evident then that
k=0
n
Weierstrass M-test. We can see though that (1 x) will
this test. Assume we
P also pass
n
pick a value x = > 0. Then we can bound the series (1 ) with some function k n
such that k = 1  > 1 for some >  > 0. We are guaranteed such a point exists, and
hence this series passes the Weierstrass M-test on all sets of the form (0, a] with a < 1.
Exercise 8.3.12. Prove that if D is a subset of Rp and {Fn } is a sequence of functions from
D to Rq , then {Fn } fails to converge uniformly to 0 if and only if there is a sequence {xn }
in D such that the sequence of numbers {Fn (xn )} does not converge to 0.
Proof. Assume that {Fn } does not converge to 0, and that there does not exist a sequence
{xn } such that {Fn (xn )} fails to converge to 0. Then for all sequences {xi } D, we find
that {Fn (xi )} 0. This means that for all x, |Fn (x)| <  for some  > 0 with x D
and n N for some N . But this is the definition of convergence for{Fn }. Hence, it cannot
be that {Fn } fails to converge provided every sequence of {xn } results in {Fn (xn )} going to 0.
Now, assume that {Fn } converges to 0, but there exists a sequence {xn } D such that
Fn (xn ) does not converge to 0. But then, it is not true that |Fn (x)| <  whenever x D
and n N for some N,  > 0. As such, {Fn } would no longer converge.
Therefore, {Fn } converges to 0 if and only if for all sequences {xn } D, {Fn (xn )}
0.

8.4. LINEAR FUNCTIONS, MATRICES

8.4

Linear Functions, Matrices

8.5

Dimension, Rank, Lines, and Planes

55

56

CHAPTER 8. FUNCTIONS ON EUCLIDEAN SPACE

Chapter 9
Differentiation in Several Variables
9.1

Partial Derivatives

9.2

The Differential

Exercise 9.2.1. If L : Rp Rp is a linear function, show that dL = L. In other words, if


L has a matrix A, then A is the differential matrix of the linear function L(x) = Ax.
h
Proof. It suffices to show that limh0 L(a+h)L(a)A
= 0. We need to then show that
|h|
L(a + h) L(a) A h. But by linearity we can write this is L(a) + L(h) L(a) A h = 0,
from which it is evident that L(a) = A h, and hence, that dL = L.

Exercise 9.2.5. Find the differential of the real-valued function f (x, y, z) = xy 2 cos(xz).
Then find the best affine approximation to f at the point (1, 1, /2).


The differential is df = xy 2 z sin (xz) + y 2 cos (xz) 2xy cos (xz) x2 y 2 sin (xz) .
We find the linear approximation of f at the given point as follows:
f (x, y, z) f (1, 1, /2) + fx (1, 1, /2)(x 1) + fy (1, 1, /2)(y 1) + fz (1, 1, /2)(z /2)




=0+
(x 1) + 0 + z +
2
2

1
= z (x 1) = (x + 2z 2) .
2
2
2
We can see that we can now approximate f by the linear function L(x, y, z) = 21 (x + 2z 2).
Evaluating L at the point of interest, we see that L(1, 1, /2) = 0.
Exercise 9.2.7. Prove that if f is a real-valued function defined on an open subinterval of
R containing a and if S is an affine function such that f (a) = S(a), and
f (a + h) S(a + h)
=0
h0
h
lim

then S(a + h) = f (a) + f 0 (a)h.


57

58

CHAPTER 9. DIFFERENTIATION IN SEVERAL VARIABLES

Proof. Let S(x) = m x + b. Then


f (a + h) S(a + h) = f (a + h) m a m h b.
Since we assume f (a) = S(a), we find then that b = f (a) m a. Then, we rewrite as
f (a + h) S(a + h) = f (a + h) f (a) m h. We find
0 = lim

h0

f (a + h) f (a)
mh
f (a + h) S(a + h)
= lim
lim
.
h0
h0
h
h
h

This implies that m = f 0 (a) and S(x) = f 0 (a)x + f (a) m a. If we let x = a + h, we find
S(a + h) = f 0 (a)(a + h) + f (a) a f 0 (a) = f (a) + f 0 (a)h.
This completes the proof.
Exercise 9.2.11. If f : Rp Rp is differentiable at a Rp , then show that, for each h Rp
the function g : R R defined by g(t) = f (a + th) has a derivative at t = 0. Can you
compute it in terms of df (a) and h?
Proof. MISSING A SOLUTION
Exercise 9.2.12. Prove that a function F : Rp Rq is affine if and only if it is differentiable
everywhere and its differential matrix is constant.
Proof. We start by proving that if a function F : Rp Rq is affine, then it is differentiable
everywhere with a constant differential matrix. This is evident when we consider that an
affine function F (x) is of the form F (x) = b + L(x), with L : Rp Rq , with an associated
matrix p q-matrix A. Also, b, x Rp . By exercise 9.2.1, we see that for a linear function
L(x) with associated matrix A, dL = A. From this we see that F is differentiable everywhere
with a constant differential matrix.
We now proceed to consider a function which is differentiable everywhere, and has a constant
differential matrix. The function F = (F1 (x), F2 (x), . . . , Fp (x)). Since the differential is
constant, we can write Fi = a1,i x1 + a2,i x2 + . . . + ap,i xp + bi for some constants a1,i , a2,i , ..., bi .
This implies that F (x) = b + Ax where A = aj,i and b = [b1 , b2 , . . . , bq ]T . Hence, F (x) is
affine.

9.3

The Chain Rule

Exercise 9.3.1. If F is a function from an open subset U of Rp to Rq which is differentiable


at a and if B is an r q matrix, then show that d(BF )(a) = BdF (a). Here, BF (x) is the
matrix B applied to the vector F (x) and BdF (a) is the product of the matrix B and the
matrix dF (a).

9.3. THE CHAIN RULE

59

Proof. F is differentiable at a if and only if there exists some function q(h) for h near 0 such
that F (a + h) F (a) = Q(h)h, and Q(0) = dF (a). Let Q be a function satisfying these
conditions. Now let B be some p q-matrix. Then the following holds:
B (F (a + h) F (a)) = B Q(h) h
B F (a + h) B F (a) = B Q(h) h
Note that BQ(h) is continuous at 0 and that B F is differentiable at a. We then have
d(B F )a = BQ(0) = B dF (a).
g
Exercise 9.3.4. If f is a differentiable function on R and g(x, y) = f (xy), show that x x

g
y y = 0.

Proof. MISSING A SOLUTION


Exercise 9.3.6. If u is a variable which is a differentiable function of (x, y) in an open set
U R2 , if x and y are differentiable function of (s, t) V for an open set V R2 , and
if (x, y) U whenever (s, t) V , then use the chain rule to obtain an expression for u
s
and u
on V in terms of the partial derivatives of u with respect to x and y and the partial
t
derivatives of x and y with respect to s and t.

d x d y
d
=
+
s
x s y s
d
d x d y
=
+
t
x t
y t
Exercise 9.3.8. If F (x, y) = (f1 (x, y), f2 (x, y)) is a differentiable function from R2 to R2
and if we define G : R2 R2 by G(s, t) = F (st, s + t), find an expression for the differential
matrix of G in terms of the partial derivatives of f1 , f2 .
Proof. MISSING A SOLUTION
Exercise 9.3.9. If (x, y, z) are the Cartesian coordinates of a point in R3 and the spherical
coordinates of the same point are r, , , then
x = r cos() sin()

y = r sin() sin()

z = r cos()

Let u be a variable which is a differentiable function of (x, y, z) on R3 . Find a formula for the
partial derivatives of u with respect to r, , in terms of its partial derivatives with respect
to x, y, z.

60

CHAPTER 9. DIFFERENTIATION IN SEVERAL VARIABLES

u
d x d y d z
=
+
+
r
x r y r z r
d
d
d
= cos() sin()
+ sin() sin()
+ cos()
x
y
z
u
d x d y d z
=
+
+

x y z
d
d
+ r sin () cos ()
= r sin () sin ()
x
y
u
d x d y d z
=
+
+

x y z
d
d
d
= r cos () cos ()
+ r sin () cos ()
r sin ()
x
y
z

9.4

Applications of the Chain Rule

Exercise 9.4.3. Find the parametric equation for the tangent line to the curve (t) =
(t3 , 1/t, e2t2 ) at the point where t = 1.

|
We note that (1) = (1, 1, 1), and d(1) = 3 1 2 . Then the tangent line (t) can
be written as


2
3
1
3t 3
3t 2
(t) = 1 + (t 1) 1 = 1 + 1 t = 2 t
1
2
1
2t 2
2t 1
Exercise 9.4.6. Show that the gradient at x Rp of the function g(x) = x x is the vector
2x.
We note that we are considering
the standard dot product as inner product, such that
P
g(x) = hx, xi = x22 + . . . x2n = ni=1 x2i . We notice that deg(x) = (2x1 , 2x2 , . . . , 2xn ) = 2x| .
Exercise 9.4.11. Find the equation of the tangent plane to the cone z = x2 + y 2 at the
point (1, 2, 5).
We see that

z
x

= 2x and

z
y

= 2y. Then:
z
z
(x x0 ) +
(y y0 )
x
y
z
z
z=
(x x0 ) +
(y y0 ) + z0
x
y
z = 2x(x x0 ) + 2y(y y0 ) + z0
z = 2x(x 1) + 2y(y 2) + 5.

z z0 =

9.5. TAYLORS FORMULA

61

Exercise 9.4.12. Show that for each point (a, b, c) on the surface x2 + y 2 + z 2 = 1, there
is a neighborhood of (a, b, c) in which the surface may be represented as a smoothly parameterized 2-surface. Hence, there is a tangent plane to this surface at every point.
Let S denote the surface given by x2 + y 2 + z 2 = 1, which is a sphere. Consider the
subsets of S given the graphs of the functions:
p
x+ (y, z) = 1 y 2 z 2
(y 2 + z 2 < 1)
p
(y 2 + z 2 < 1)
x (y, z) = 1 y 2 z 2

y+ (x, z) = 1 x2 z 2
(x2 + z 2 < 1)

y (x, z) = 1 x2 z 2
(x2 + z 2 < 1)
p
(x2 + y 2 < 1)
z+ (x, y) = 1 x2 y 2
p
z (x, y) = 1 x2 y 2
(x2 + y 2 < 1)
Each of these functions is injective and differentiable. For any a S, we have that a is in
the image of one of these functions, so we can construct a tangent plane to thee surface at
any possible point.
Exercise 9.4.13. Find an equation for the tangent plane to the surface x2 + y 2 z 2 = 1 at
each point on the surface.
Let w = x2 + y 2 + z 2 . Then w = (2x, 2y, 2z), which means at point A = (a, b, c) we
have w(A) = (2a, 2b, 2c). Then we get the following equation:
2a(x a) + 2b(y b) + 2c(z c) = 0
2a2 + 2ax 2b2 + 2by 2c2 + 2cz = 0
2ax + 2by + 2cz = 2a2 + 2b2 + 2c2
ax + by + cz = a2 + b2 + c2 .

9.5

Taylors Formula

Exercise 9.5.1. Find the degree n = 2 Taylor formula for f (x, y) = x2 + xy at the point
a = (1, 2).
We begin by finding the necessary partial derivatives.
f
= 2x + y,
x
f
= x,
y
f
2f
=
= 1.
xy
yx

2f
=2
x2
2f
=1
y 2

62

CHAPTER 9. DIFFERENTIATION IN SEVERAL VARIABLES

We now use the known equation with these derivatives at the point a = (1, 2).




f (a, b) f (a, b)
1 2 f (a, b) 2
2 f (a, b) 2
2 f (a, b)
f (a + x, b + y) f (a, b)
+
y +
xy +
x +2
y
x
y
2
x2
xy
y 2

1
= 3 + (4x + y) +
2x2 + 2xy + y
2

1
2x2 + 2xy + 8x + y 2 + 2y + 6 .
=
2
Exercise 9.5.4. Suppose U is an open convex set and f is a differentiable real-valued
function on U . If there is a number M > 0 such that |df (x)| M for all x U , then
|f (x) f (y) M |x y|
for all x, y U .
Proof. Let x, y U . Then by Taylors theorem we have f (x) = f (y) + df (c)(x y) for some
c [y, x]. Then by algebra:
|f (x) f (y)| = |df (c)(x y)|
|df (c)||x y|
m|x y|.
This completes the proof.
Exercise 9.5.6. Show that the following form of the Mean Value Theorem is not true: if
F : R2 R2 is a differentiable function and a, b R2 , then there is a c on the line segment
joining a to b such that F (b) F (a) = dF (c)(b a). The problem here is that F is vectorvalued, not real valued.
If the function F is vector-valued as above, we cannot make sense of the equation. Specifically, we find that dF (c)(b a) is defined as a scalar multiplication. In this case, the result
is a scalar, whereas F (b) F (a) would be a case of vector addition. A scalar cannot equal
a vector. Alternatively, we could attempt to solve this issue by redefining dF (c)(b a) as
matrix multiplication. However, then we could only have equality in the case of dF begin a
square matrix; otherwise, the dimensions wouldnt match up. As such, it appears as though
we cant make sense of the equation as is for vector valued functions.
Exercise 9.5.8. Find all points of relative maximum and relative minimum and all saddle
points for f (x, y) = 1 2x2 2xy y 2 .


f
We begin by finding the differential f (x, y) = f
,
= (4x 2y, 2x 2y). This
x y
is only zero at the point (0, 0). We calculate the Discriminant at this point:
D = (4)(2) (2)2 = 8 4 = 4.
Since D > 0,

2f
x2

< 0 we find that we are dealing with a point of local maximum.

9.6. THE INVERSE FUNCTION THEOREM

63

Exercise 9.5.9. Find all points of relative maximum and relative minimum and all saddle
points for f (x, y) = y 3 + y 2 + x2 2xy 3y.
We begin by taking the various partial derivatives:
2f
=2
x2
2f
= 6y + 2
y 2

f
= 2x 2y,
x
f
= 3y 2 + 2y 2x 3,
y
2f
2f
=
= 2
xy
yx

We then find the points where f = ~0. As f = (2x 2y, 3y 2 + 2y 2x 3), we find that
we have critical points at (1, 1) and (1, 1).
At the point (1, 1) we have D = 2(6(1) + 2) (2)2 = 12, and therefore we are
dealing with a saddle point here.
At the point (1, 1) we have D = 2(6 + 2) (2)2 = 4, and hence we have a point of local
minimum here.

9.6

The Inverse Function Theorem

Exercise 9.6.2. Show that the function F : R2 R2 defined by F (x, y) = (x2 + y 2 , xy)
has a smooth local inverse near points (x, y) where x 6= y. Find the inverse function F 1
on the set {(x, y) : x < y < x} and identify its domain. Calculate the differential of this
inverse function (1) directly and (2) by using the Inverse Function Theorem. Verify that the
two methods give the same answer.
Let u = x2 + y 2 and v = xy. Then we find that u + 2v = (x + y)2 and u 2v = (x y)2 .
On the set {(x, y) : x < y < x} we find that x + y > 0 and x y > 0. This means that
JF (x, y) = 2(x2 y 2 ) 6= 0 and we have an inverse. We find then that
x+y =

xy =

u + 2v

u 2v

From this we get that




1 
u + 2v + u 2v
2


1 
y=
u + 2v u 2v
2

x=

We calculate the differential:


 x
u
y
u

x
v
y
v

"


=

1
4 u+2v
1
4 u+2v

1
4 u2v
1
4 u2v

1
2 u+2v
1
2 u+2v

1
2 u2v
1
2 u2v

64

CHAPTER 9. DIFFERENTIATION IN SEVERAL VARIABLES

Now according to the inverse function theorem:


dF 1 (b) = (dF (a))1
" 2 2
=

(x2 +y 2
y
(xy)
y
1

(x +y )
x
(xy)
x

#!1


2x 2y
=
y x

y 
x
x2 y
2
2x2 2y 2
=
y
x
2x2 2y
2
x2 y 2
"
1
1

1
+ 4u2v

4 u+2v
2 u+2v
= 1
1
1

4 u2v 2 u+2v +
4 u+2v

and substituting:
1
2 u2v
1
2 u2v

Exercise 9.6.5. Find a smooth local inverse function near (1, /2) for the function F of
Example 9.6.6.


cos() r sin()
We find that dF (r, ) =
. The determinant is non-zero whenever
sin() r cos(theta)
r 6= 0. At the point F (a) = (0, 1). We know by the inverse function theorem that dF 1 (b) =
(dF (a))1 . We now use this information and find the inverse differential:
dF

(b) = (dF (a))

 

cos()
sin()
0 1
=
=
.
r1 sin() r1 cos()
1 0

We find that the inverse function is F 1 (x, y) =

p

x2 + y 2 , tan1 (y/x) .

Exercise 9.6.8. Show by example that the result of the previous problem is not true if
U is only assumed to be connected, rather than convex. Hint: Try the function F (x, y) =
(x2 y 2 , 2xy) on R2 \{0}.


2x
2y
2
2
Consider the function f (x, y) = (x y , 2xy). Then df =
. Then the
2y
2x
determinant of df is greater than 0, namely it is 4(x2 + y 2 ). Consider now U = R2 \{(0, 0)},
which is a connected but not a convex set. Here, df is non-singular but f is not injective
since f (1, 1) = f (1, 1).
Exercise 9.6.10. Show that the condition that dF (a) be non-singular is necessary in the
Inverse Function Theorem by showing that if a function F from a neighborhood of a in RP
to Rp is differentiable at a and has an inverse function at a which is differentiable at F (a),
then dF (a) is non-singular.
We are trying to show that if F is locally invertible, its differential matrix is also invertible,
i.e. non-singular.

9.7. THE IMPLICIT FUNCTION THEOREM

65

Assume the inverse of F exists, and refer to it as F 1 , such that F F 1 = I. If we


differentiate, we find that we are looking for a function such that
I = dF dF 1 (F ) .
By linear algebra, this implies that (dF (a))1 is the inverse differential of dF (a), which
implies that dF (a) is non-singular.
Exercise 9.6.12. If F : Rp Rp is a C 1 function, what can you say about F at a point of
Rp where |F | has a local minimum? How about a point where |F | has a local maximum?
p
The norm of a function F is defined as |F | = (F1 )2 + . . . + (Fp )2 for a function with
p-components. Differentiating both sides with respect to some variable x yields the equation
2|F |

d|F |
dF1
dFp
= 2F1
+ . . . + 2Fp
x
dx
dx

We note that since we are dealing with local minima/maxima, that


get the equation
p
X
dFi
0=
.
Fi
dx
i=1

d|F |
x

= 0. As such, we

This is an interesting equation.

9.7

The Implicit Function Theorem

Exercise 9.7.1. Are there any points on the graph of the equation x3 + 3xy + 2y 3 = 1
where it may not be possible to solve for y as a smooth function of x in some neighborhood
of the point?
Consider the function f (x, y) = x3 + 3xy 2 + 2y 3 1. By the implicit function theorem,
6= 0. We find that
we can solve this function for y as a smooth function of x whenever f
y
f
2
= 6xy + 6y . Solving this for zero is equivalent to solving y(x + y) = 0. Therefore, we
y
can solve for y as a smooth function of x at all points where y 6= 0 and y 6= x.
Exercise 9.7.3. Find

(f1 ,f2 )
(u,v)

if

f1 (x, y, u, v) = u2 + v 2 + x2 + y 2 ,
f2 (x, y, u, v) = xu + yv + x y.
At which points (x, y, u, v) is this matrix non-singular?
We solve for the differential:
(f1 , f2 )
=
(u, v)

 f1
u
f2
u

f1
v
f2
v

2u 2v
=
x y

We find that this matrix is singular whenever 2uv = 2vx. Hence, this matrix is non-singular
for all points (x, y, u, v) S = {(x, y, u, v) : 2uv 6= 2vx}.

66

CHAPTER 9. DIFFERENTIATION IN SEVERAL VARIABLES

Exercise 9.7.4. Show that the system of equations


u2 + v 2 + 2u xy + z = 0
u3 + sin(v) xu + yv + z 2 = 0
has a solution for (u, v) as a smooth function of (x, y, z), in some neighborhood of (0, 0, 0),
with the property that (u, v) = (0, 0) when (x, y, z) = (0, 0, 0).
Let f1 = u2 + v 2 + 2u xy + z and f2 = u3 + sin(v) xu + yv + z 2 . We solve for the
differential at (0, 0, 0, 0, 0):


f
2u + 2
2v
=
3u2 x cos(v) + y
(u, v)


f
2 0
(0, 0, 0, 0, 0) =
0 1
(u, v)
As the last differential is if non-singular, this is possible.
Exercise 9.7.6. For the equation xy + yz + xz = 1, at which points on the solution set S
is there a neighborhood in which S is a smooth 2-surface? At each such point (a, b, c), find
an equation of the tangent plane.
This function describes a two sheeted hyperbola. Let F (x, y, z) : xy + yz + xz = 1.
THen F = (y + z, x + z, y + x). We find that F = (0, 0, 0) only at the points (0, 0, 0)
and (x, x, 0). Of these, the only points on the graph are (1, 1, 0) and (1, 1, 0). We find
that F (1, 1, 0) = F (1, 1, 0). This means that there exists a neighbord V of (1, 1, 0) and
(1, 1, 0) with V R3 there exists a level set S = {u V : F (u) = 0} which is a smooth
p-surface. The tangent space at (1, 1, 0) and (1, 1, 0) consists of the set of the solutions
u to F (1, 1, 0)(u (1, 1, 0)) = 0 and F (1, 1, 0)(u (1, 1, 0)) = 0.
Exercise 9.7.8. For the system of equations
x2 + y 2 + u2 3v = 1,
2x + xy y + 3u2 9v = 0,
find all points on the solution set S for which there is a neighborhood in which S is a smooth
2-surface.
Let f1 = x2 + y 2 + u2 3v 1 and f2 = 2x + xy y + 3u2 9v. We find the differential
"
#
f1
f1
f1
f1
(f1 , f2 )
x
y
u
v
= f
f2
f2
f2
2
(x, y, u, v)
x
y
u
v


2x
2y
2u 3
=
2 + y x 1 6u 9
From this we can see that wepcan express the system of equations in terms of x, y as functions
of u, v provided that x 6= (y + 1)2 .

Chapter 10
Integration in Several Variables
10.1

Integration over a Rectangle

10.2

Jordan Regions

10.3

The Integral over a Jordan Region

10.4

Iterated Integrals

10.5

The Change of Variables Formula

67

68

CHAPTER 10. INTEGRATION IN SEVERAL VARIABLES

Chapter 11
Vector Calculus
11.1

1-forms and Path Integrals

Exercise 11.1.1. Find a smooth curve in R2 which traces the straight line from (1, 2) to
(3, 0).
The solution is the function (t) = ~u + t(~v ~u) where ~u = (1, 2), ~v = (3, 0), and t [0, 1].
We rewrite:
 
   
1
3
1
(t) =
+t

2
0
2
 
  

1
2
1 + 2t
=
+t
=
2
2
2 2t
Exercise 11.1.2. We find the derivative 0 (t) = (cos(t) t sin(t), sin(t) + t cos(t)). We
proceed to find the norm
| 0 (t)| =

(cos(t) t sin(t))2 + (sin(t) + t cos(t))2 = 1 + t2 .

To find the length, we solve the integral


t = tan().
Z
0

1 + t2 dt =

R 4
0

1 + t2 dt. We utilize a trig substitution, lettting

tan1 (4)

sec()d

0
1

(4
= ln (sec() + tan())|tan
)
0





1
2
= ln sec tan (4) + 4 = ln 4 + 16 + 1

The last step is justified by letting sec (tan1 (4)) = sec(), and solving tan2 ()+1 = sec2 ().
69

70

CHAPTER 11. VECTOR CALCULUS

11.2

Change of Variables

11.3

Differential Forms of Higher Order

11.4

Greens Theorem

11.5

Surface Integrals and Stokes Theorem

11.6

Gausss Theorem

11.7

Chains and Cycles

You might also like